FOUNDATIONS LECTURE FINAL

¡Supera tus tareas y exámenes ahora con Quizwiz!

A nurse must give 1 g of cephalexin, PO, q 6 hr × 3 days. The supply on hand is 500 mg/capsule. How many capsules should the nurse administer for each dose?

Answer: 2 CAPSULES PER DOSE

What is a purpose of a hand-off report? Ensures continuity of care and patient safety Keeps the doctor informed Completed when a patient is discharged to home Determines patient assignments

Answer: a A hand-off report shares patient-specific information from one caregiver to another or among interprofessional team members to ensure continuity of care and patient safety. The hand-off report is usually shared among direct caregivers. Doctors are kept informed verbally by the nurse, through SBAR reporting, and by accessing the EHR. A patient being discharged to home is given discharge instructions but is not being "handed off" to another caregiver; thus, a hand-off report is not appropriate. Patient assignments are determined before a hand-off report is needed and are based on patient acuity, staffing, and a number of other factors. LO: 10.5

From the nurse's knowledge about the emerging adult according to Arnett's theory, which behavior by a 21-year-old hospitalized male patient is most appropriate for his age group? a. Talking about college courses that he is taking while working part-time at a restaurant b. Requesting that his mom be present when his IV line is started c. Stating that he cares for his disabled father and his 2-year-old daughter d. Becoming upset that he is not giving back to his community

Answer: a According to Arnett (2006), emerging adults are shifting their choices, and many are unstable; they explore more possibilities and move gradually toward enduring choices. The young adult is often slow in transitioning due to pursuing higher education. This 21-year-old is pursuing education while working part-time, which fits within Arnett's theory for this age group. A 21-year-old male would be exhibiting regression if he needed his mom with him during an intravenous (IV) start. Caring for his father and daughter would be more typical of middle adults, who are called the sandwich generation. Giving back to one's community is generativity, which is Erikson's middle adult developmental task. LO: 18.3

What is the most significant problem that may result from improperly written NANDA-I nursing diagnostic statements? a. Lack of direction for formulating patient plans of care b. Omission of physician or primary care provider orders c. Combining of two unrelated patient concerns d. Increased team collaboration needs

Answer: a Accurate nursing diagnostic statements provide direction for the development of individualized plans of care. Orders are part of the patient's assessment data. Combining unrelated patient problems is a function of diagnostic development, not a result of an improperly written statement. Poorly written nursing diagnostic statements may or may not result in increased team collaboration. LO: 7.6

. The nurse recognizes which cue from the patient as a physiologic response to acute pain? a. Increased blood pressure b. Decreased pulse c. Increased temperature d. Restlessness

Answer: a Acute pain can increase blood pressure and pulse rate but may not affect temperature. Restlessness is a psychological response, not physiologic. LO: 36.5

The nurse providing education to a patient in the sleep lab understands that the most common type of sleep apnea is caused by which of these factors? a. Airway collapse b. Lack of exercise c. Dietary factors d. Medication use

Answer: a Airway collapse of the soft structures of the upper airway is the most common cause of sleep apnea, which is called obstructive sleep apnea. Lack of exercise and certain dietary factors may adversely affect sleep patterns, especially the initiation of sleep. Medications in the proper dose seldom cause sleep apnea unless improperly combined with other medications or taken with alcohol.

Culturally competent care would encourage which action by a patient's family? a. Asking the family's spiritual advisor to visit the patient b. Speaking English to everyone involved in patient care c. Adhering to highly publicized restrictive unit visiting hours d. Limiting food consumption to items provided by the cafeteria

Answer: a Allowing a patient to meet with a spiritual advisor recognizes the importance of a patient's spiritual needs. Culturally competent care allows for flexibility within safety guidelines and patient care limitations. Limiting language use, food consumption, and visiting hours in a strict manner without sensitivity to a patient's preference do not reflect culturally competent nursing care.

What aspect of culture is a full-time employed granddaughter of an elderly female exhibiting if she asks the social worker to place her grandmother in an extended-care facility against the wishes of her parents? a. System change b. Gender role c. Cultural norms d. Shared attributes

Answer: a As one aspect of a society changes, the systems within that society change. The granddaughter in this case is employed full-time and unable to adequately care for her grandmother at home. Therefore, her request to have the grandmother placed in an extended-care facility reflects societal changes that affect traditional culture expectations of one generation providing care in the home. Shared attributes, cultural norms, and gender role are all challenged by the granddaughter's actions.

The nurse has been caring for a patient who just died. The patient's daughter is crying uncontrollably, saying, "She was my best friend. I thought she would make it! I don't know what I am going to do." What is the nurse's best response? a. Express sympathy and ask if she would like to talk with a chaplain. b. Give the daughter time to cry in her mother's room alone. c. Ask the daughter if her father is still living. d. Inquire if the daughter would like to pray.

Answer: a At crisis times, spiritual advisers or chaplains are the best resource with the expertise to address family members' needs. Leaving the daughter alone or inquiring about her father would not provide the emotional support needed. The nurse should avoid making suggestions to the daughter but instead should seek to identify needs from the daughter's cues. LO: 22.3

Three weeks after delivery of her baby, a patient started a diet of 800 calories per day and started jogging 2 miles twice per day. The nurse recognizes that the patient's behavior may be most influenced by which motivating factor? a. Body image b. Environment c. Illness behavior d. Chronic illness

Answer: a Body image is a self-ascribed attribute that influences a person's ability and desire to change, and it may be a motivating factor to maintain the change. Self-concept is a general and broader factor that motivates less specific responses. The environment is not an influencing factor because this is a personal and possibly self-destructive behavior pattern for weight loss. Pregnancy is not a chronic illness. LO: 16.6

When the nurse is establishing goals for a community health initiative, which strategy is most important to incorporate in the planning process? a. Collaboration with key stakeholders b. Help from professional interpreters c. Location of schools and businesses d. Gender of primary care providers

Answer: a Collaboration with key stakeholders is critical to effective goal planning in a community. Valuing the ideas of stakeholders increases support for initiatives and has the potential to increase participation and improve outcomes. Help from professional interpreters, the location of schools and businesses, and the gender of health care providers are factors to be considered after goals are established and interventions are being considered for a specific target population. LO: 23.7

Teaching a patient to use an incentive spirometer by demonstration, with a return demonstration by the patient, is an example of teaching based on which domain of learning? Psychomotor Affective Psychosocial Cognitive

Answer: a Demonstration along with a return demonstration by the patient is an example of psychomotor domain learning. Affective domain learning integrates new knowledge by recognizing an emotional component. Psychosocial is not one of the domains of learning. Cognitive domain learning is based on knowledge and material that is remembered, memorized, and recalled. LO: 14.4

Which nursing action is appropriate for a patient with sensory overload? a. Dimming the lights b. Performing care a little at a time c. Leaving the patient's door open d. Rushing to get care done quickly

Answer: a Dimming the lights decreases sensory stimuli which alleviates sensory overload. Constant disruption adds to the overload, as does leaving the door open and rushing while in the room. A calm, quiet atmosphere diminishes the overload. LO: 31.6

The nurse provided preoperative teaching about pain management to a patient scheduled for surgery. Which postoperative activity by the patient indicates the effectiveness of teaching? a. Doing something enjoyable, such as relaxing and reading a book b. Requesting pain medication when no longer able to tolerate the pain c. Removing the postoperative dressing to see the surgical incision d. Refusing to wear antiembolism stockings while still on bed rest

Answer: a Distraction is a nonpharmacologic complementary therapy technique that is used to assist with pain control. A delay in requesting pain medication makes it difficult to break the pain cycle and get the pain under control quickly. Removing the dressing increases the risk of infection and injury. Refusing to wear antiembolism stockings increases the risk of clot formation. LO: 37.8

When administering analgesics to elderly patients, what information does the nurse need to understand? a. Start with a low dose, and increase the dose as needed for pain relief. b. Start with a high dose and decrease the dose as pain is relieved. c. Start with a midrange dose and increase or decrease the dose as needed for pain. d. Start with a low dose and decrease the dose every other day.

Answer: a Due to decreased metabolism and clearance of medications, start with a lower dose and increase as indicated for pain relief. A high dose may result in drug toxicity. Too low of a dose will not relieve pain. LO: 36.3

The nurse is developing a plan of care for a patient. What is the most appropriate goal for a patient related to medications? a. The patient will administer all medications correctly by discharge. b. The patient will be taught common side effects of prescribed medications. c. The patient will have a good understanding of prescribed medications. d. The patient will have all medications administered by staff as prescribed.

Answer: a Evaluating the patient's ability to correctly administer medications is a patient-centered and measurable goal. Teaching side effects and administering medications by staff are nursing goals. Desiring a "good understanding" is not a measurable goal. LO: 35.8

Which nursing diagnosis is most appropriate for a patient with expressive aphasia? a. Impaired Verbal Communication b. Acute Confusion c. Self-Care Deficit d. Impaired Mobility

Answer: a Expressive aphasia occurs when people are not able to express themselves with words. They might be able to understand what is being said but not able to respond appropriately. Therefore, Impaired Verbal Communication is most appropriate. The nursing diagnosis of Acute Confusion is appropriate for patients who are not oriented to person, place, time, or situation. The diagnosis of Self-Care Deficit would apply to a patient who cannot independently take care of activities of daily living (ADLs). Patients with limitations of movement may have a nursing diagnosis of Impaired Mobility. LO: 31.4

What action by the nurse would most ensure accurate interpretation of patient communication? a. Providing feedback regarding the conveyed message b. Writing down the patient's conversational highlights c. Assuming significant cultural differences exist d. Verifying the patient's emotional state

Answer: a Feedback is the most effective way to avoid misinterpretation of a message. It helps ensure that the message sent is perceived by the receiver in a way that is consistent with the intention of the sender. Writing down conversational highlights is a form of documentation that can still be misinterpreted unless feedback is sought. Avoid making assumptions regarding cultural differences. Verifying a patient's emotional state provides insight into a patient's state of mind, but it does not ensure accurate interpretation of a conversation. LO: 3.1

The nurse is orienting new staff to a clinical unit that provides palliative care. A new employee asks what "grief" is exactly. Which statement indicates that the nurse has correctly defined grief? a. The emotional response to a loss b. The outward, social expression of a loss c. The depression felt after a loss d. The loss of a possession or loved one

Answer: a Grief is the emotional response to a loss that is an individualized and deeply personal feeling caused by a real or perceived loss. The outward, social expression of a loss is bereavement. Depression is not a normal response to loss, although there are many emotional feelings that occur due to a loss. The loss of a possession or a loved one is considered an actual loss. LO: 42.1

Which short-term goal would be most appropriate for a patient with the nursing diagnosis Anxiety with supporting data, including upcoming diagnostic tests, expressions of concern, and pacing around the room? a. Patient will discuss specific aspects of concern. b. Nurse will administer prescribed antianxiety medication. c. Patient will understand diagnostic test procedures. d. Nurse will describe test procedures in detail to allay concerns.

Answer: a Having the patient discuss specific aspects of concern allows the nurse the opportunity to assess the patient's level of anxiety and what interventions might be most appropriate to help allay the stated concerns. Goals must be patient centered, measurable, and realistic. None of the other three goals meet these criteria. Two of the goals are nurse focused. The action "understand" is not measurable. LO: 32.6

A patient who has a serious back injury received intravenous medication for pain approximately 1 hour earlier. The patient practices relaxation techniques but still is reporting pain at a level of 9 of 10. What is the next action that should be taken by the nurse? a. Report the lack of pain relief to the primary care provider. b. Tell the patient to give the medication more time. c. Reposition the patient and try diversion activities. d. Document in the nurse's notes that the patient has a low pain tolerance.

Answer: a If the patient with a serious injury is not obtaining pain relief from pharmacologic and nonpharmacologic interventions, the primary care provider should be notified. Waiting longer and using more nonpharmacologic interventions are not likely to relieve pain in this situation. LO: 36.8

The nurse researcher provides participants with informed consent so that what ethical principle is upheld? a. Respect for persons b. Beneficence c. Justice d. Ethics

Answer: a Informed consent abides by the principle of respect for persons. The beneficence principle reminds researchers to minimize harm and maximize benefits. The principle of justice requires researchers to treat people fairly and to design research to select subjects equitably and avoid exploitation of vulnerable populations. Ethics is a general concept, not a specific principle. LO: 13.3

. What strategy would be most effective in communicating with a highly anxious adult immediately before surgery? a. Providing specific, concise instructions b. Detailing likely causes of the patient's anxiety c. Focusing on postoperative details d. Using instructional multimedia DVDs

Answer: a Only essential information supplied in short, succinct sentences can be comprehended by adults who are extremely anxious. The source of this patient's anxiety is already stated to be the surgery; thus, the nurse need not elaborate on it. Postoperative teaching is best completed well in advance of surgery and reinforced after completion of the procedure. Multimedia DVDs are not effective teaching tools immediately before surgery. They may be helpful for a patient to watch at least 24 hours before a scheduled procedure to allow time for elaboration on topics not totally understood by the patient. Nurses must always check with the patient to verify that critical information is understood regardless of what form of communication has been used. LO: 3.5

A famous rock star has just been admitted to Unit 12A after an automobile accident. A nurse on Unit 12B who is a fan of the musician uses the electronic health record (EHR) to find out how the patient is doing. Which is true regarding the use of a patient's EHR? Only staff caring for the patient should access this record. Permission from a supervisor is needed to read this record. The patient's record can be discussed with the nurse's co-worker. The nurse can call a friend who works at the local newspaper.

Answer: a Patient information should be accessed only by staff caring for that patient. Security codes are required for EHR access, and access of records can be monitored. Professional ethics should dictate the nurse's behavior, and only the records of patients being cared for should be accessed. A patient's record can be discussed only with those who are caring for the patient, and, because the nurse is not on the same unit, the records should not be accessed or discussed by that nurse. Health Insurance Portability and Accountability Act (HIPAA) laws prevent the discussion of private patient information with anyone outside of the team providing care. LO: 15.7

The patient who had a below-the-knee amputation 3 days ago complains of pain from the amputated extremity. Which response by the nurse best explains what the patient is experiencing? a. "Your phantom pain will subside when the brain realizes the lower extremity is no longer there." b. "Your radiating pain will continue for months because the lower extremity is no longer there." c. "You are suffering from referred pain, which you will always have, but it will lessen with time." d. "You are experiencing psychogenic pain because loss of an extremity is an emotional loss."

Answer: a Phantom pain occurs when the brain continues to receive messages from an area of amputation. Over time, the brain will adapt to the loss of the limb. Radiating pain extends from the source of pain to an adjacent area of the body. Referred pain originates in one area of the body but hurts in another area of the body. Psychogenic pain is pain perceived by the patient but has no physical pain. LO: 36.3

8. The endocrine system releases excessive hormones during episodes of acute pain. The nurse should monitor patients experiencing acute pain for which potential problem? a. Hyperglycemia b. Migraine headache c. Hypokalemia d. Diarrhea

Answer: a Release of hormones causes the blood glucose level to increase, causing hyperglycemia. Hypokalemia may result from the metabolic effects of genitourinary injury. Constipation results from decreased intestinal motility. Migraine headaches are not a result of hormone release during acute pain. LO: 36.3

A male patient is told that he may have colon cancer. Which response by the patient best indicates that his initial appraisal of the situation is that it is primarily a challenge to be met? a. Requesting information on various treatment options b. Demanding to see another physician immediately c. Storming out of the gastroenterologist's office d. Yelling at the nurse who is scheduling his colonoscopy

Answer: a Requesting further information regarding treatment indicates that the patient is viewing the situation primarily as a challenge to be faced. Primary and secondary appraisals determine whether the stressful situation or transaction is a threat or a challenge. A threat invokes the possibility of harm or loss, whereas a challenge holds the possibility of benefit. Demanding, angry behavior indicates that the patient feels threatened rather than challenged. LO: 32.1

When caring for a hearing-impaired patient, use of which action by the nurse would facilitate communication? a. Speaking clearly with distinct words b. Talking slowly to facilitate understanding c. Sitting behind the patient to decrease distractions d. Standing near the patient's affected ear to balance sound

Answer: a Speaking clearly without shouting facilitates communication with the hearing-impaired patient by giving each word separate emphasis. Talking distinctly, but not too slowly, and allowing the patient to see facial expressions and read lips, with the use of hearing aids if prescribed, are good communication techniques. Speaking into the ear with the better hearing is recommended. LO: 31.6

A teenage girl faces a long hospitalization after surgery. How can the girl's continued development be fostered? a. Encourage her to write her feelings in a journal. b. Divert her attention by playing video games. c. Encourage her to work on craft projects. d. Make sure her parents are constantly by her side.

Answer: a Teenagers are capable of abstract thought and often find it helpful to verbalize their feelings either by talking or by journaling. Diverting her attention by playing video games and encouraging her to work on craft projects are measures that may be used to help her pass the time but are not methods of fostering teen development. Teens usually do not wish parents to be at their side constantly and may want some private time away from them. LO: 17.1

Which activity best illustrates the use of the Health Promotion Model (HPM) by the nurse to increase the level of well-being for a patient immediately after surgery? a. Holding a pillow across his chest when coughing and deep breathing b. Encouraging the patient to eat his entire evening meal c. Changing his surgical dressing daily as ordered by the physician d. Asking his family to step out of the room during dressing changes

Answer: a The HPM describes an individual's interaction with his environment as he engages in behaviors that promote health. The patient using a pillow as a splint is interacting with his environment to prevent atelectasis and infection. LO: 16.2

A nursing student is taking care of a patient with possible appendicitis and is curious about the best method of diagnosing this disorder. What does the nursing student have to consider in formulating a question using the PICO format (patient, population, or problem; intervention; comparison intervention; outcomes) to determine evidence-based practice (EBP)? a. Problem, appendicitis; interventions, ultrasound versus computed tomography (CT) scan; outcome, diagnosis of appendicitis b. Problem, pain; interventions, hydromorphone versus morphine; outcome, pain free c. Problem, fever; interventions, cooling measures versus antipyretics; outcome, normal temperature d. Problem, appendicitis; interventions, complete blood count versus chemistry panel tests; outcome, painless

Answer: a The PICO format asks for the patient's problem, the two possible interventions for comparison (i.e., diagnostics), and the desired outcome from the interventions. LO: 13.5

Which note is an example of the S in SBAR? Patient resting; pain was rated 3/10 1 hour after receiving narcotic analgesic. Patient was admitted on evening shift with a fractured right femur after a fall at home. Patient's pain was rated 8/10 before administration of narcotic pain medication. Assess pain every 2 hours, continue pain medication as prescribed, and provide backrub.

Answer: a The S in SBAR stands for situation. In this case, the patient is resting, and the pain is rated 3/10 at 1 hour after receiving a narcotic analgesic. Describing the admission reason and time provides the background (B). Assessment (A) of this patient revealed pain rated 8/10 before giving pain medication. The nurse's recommendation (R) is that pain should be assessed every 2 hours and that pain medications should be given as prescribed. LO: 10.5

Which action by a 3-month-old infant would the nurse interpret as an example of Piaget's sensorimotor stage of primary circular reaction? a. Deliberately placing the thumb into the mouth b. Accidentally kicking a ball c. Searching for an object under a blanket d. Shaking a rattle

Answer: a The deliberate action of placing the thumb in the mouth elicits a pleasurable effect. Accidentally kicking a ball is not a deliberate action. Searching for objects and shaking a rattle occur at later stages. LO: 17.1

On which ethical theory do nurses implement their care when they act on the basis of the needs of one specific patient rather than the potential consequences to other patients? a. Deontology b. Autonomy c. Utilitarianism d. Nonmaleficence

Answer: a The ethical theory of deontology focuses on the act rather than on the consequences. Autonomy is an ethical concept that values an individual's right to make personal decisions. Utilitarianism is an ethical theory that focuses on the consequences of an action and the good of many rather than of an individual. The ethical principle of nonmaleficence asserts an obligation to "do no harm." LO: 11.1

A nurse providing preventive care to an overweight female with a family history of diabetes should engage in which priority care-planning activity for this patient? a. Calculating the patient's waist-to-hip ratio and recommending a daily exercise routine b. Instructing the patient to perform blood glucose monitoring once daily c. Giving the patient a month's supply of insulin needles and syringes d. Suggesting the patient participate in diabetes education classes offered at a local health facility

Answer: a The female patient does not have diabetes but is overweight and at risk due to a family history of diabetes. The best way to prevent diabetes is to keep her waist-to-hip ratio in the low-risk range (0.80 or lower). Beginning an exercise program with walking and progressing as tolerated increases muscle mass, improves depression, and strengthens the heart. LO: 16.4

A registered nurse (RN) delegates a bed bath to unlicensed assistive personnel (UAP). After lunch, the patient rings for the nurse and complains that he has not yet been cleaned up. He is very upset and angry. What should the nurse's next action be? Assist the patient in getting cleaned up. Write up the UAP for not carrying out the assignment. Report the UAP to the unit manager. Go find the UAP, and tell her to complete the bath immediately.

Answer: a The goal of delegation of any assignment is to provide efficient, patient-centered care. In this case, the patient is angry and upset—the nurse should first see to the patient's needs and address the issue with the UAP after the situation is resolved. LO: 12.5

Which activity by a unit nurse demonstrates information literacy? Researching a patient's diagnosis online Entering patient data into the electronic health record (EHR) Organizing patient data to study trends Learning a new electronic health record system

Answer: a The nurse is demonstrating information literacy (the ability to recognize when information is needed and to locate and use that information) when researching a patient's diagnosis online. Entering patient data into the EHR or learning a new EHR system demonstrates technical nursing informatics competency. Organizing patient data to study trends demonstrates a leadership level of nursing informatics competency. LO: 15.4

The nurse is in a patient room ready to administer a new medication to the patient. Which action best demonstrates awareness of safe, skilled nursing practice? a. Identify the patient by comparing the patient's name and birth date to the medication administration record (MAR). b. Determine whether the medication and dose are appropriate for the patient. c. Make sure that the medication is in the medication cart. d. Check the accuracy of the dose with another nurse.

Answer: a The right patient is one of the six rights to ensure safe administration of the medication. At least two patient identifiers should be used and compared with the armband or MAR. Determining the appropriateness of a medication for a patient should be completed prior to entering the patient room. Not all medications require another nurse to check the accuracy of the dose. Determining whether the medication is available is a time management issue. LO: 35.5

Which set of patient cues assists the nurse in determining whether the nursing actions taken to prevent airway obstruction have been effective? a. Temperature 97.8° F, breathing regular and unlabored, no cough b. Intake equals output, denies pain or chest discomfort c. Oxygen saturation 91%, shortness of breath, R 26 d. Oxygen saturation 89%, breathing shallow and regular, R 24

Answer: a These findings indicate that there is no respiratory distress. Intake and output values do not directly contribute to or affect evaluation of an airway obstruction. Decreased oxygen saturation, shortness of breath, and tachypnea can indicate that the respiratory system has been compromised, which can cause an airway obstruction. Decreased oxygen saturation, shallow breathing, and tachypnea can indicate respiratory complications. LO: 37.8

A mother brings her toddler for a well-child checkup and mentions that she is having a lot of trouble getting the child to go to bed. The nurse understands that which interventions would promote good sleep habits? a. Establish and maintain a consistent bedtime routine. b. Put the child to bed immediately after the evening meal. c. Allow the child to stay up as long as desired to increase sleepiness. d. Allow the child to sleep with the parents until the child is older.

Answer: a Toddlers and preschoolers benefit from a consistent routine to help their sleep patterns. Putting the child to bed too early (right after a meal) will not help sleep; any bedtime snacks should be a light snack containing carbohydrates. The child will become too tired if allowed to stay up as long as desired, with consequent sleep disruption the next day. The American Academy of Pediatrics does not recommend that children sleep with parents. LO: 33.6

What is an advantage of the use of electronic medical records? Electronic health records (EHRs) are always available to all health care team members. Documentation in the EHR is often illegible, causing medication and treatment errors. Patient information from the EHR cannot be shared with other departments or facilities which protects the patient's privacy. Recording in the EHR does not require any specialized training.

Answer: a Electronic health records (EHRs) are available to all health care team members. Since documentation on the EHR is computerized, it is always legible. EHR technology enables the sharing of data throughout an institution or hospital system on a need-to-know basis, which improves continuity of care, enables research, and expedites reporting of required statistics. Nurses need specialized training to begin using an EHR, or when additions or updates are added to the EHR. LO: 10.2

1. When assessing the patient for pain, which factors should the nurse consider? (Select all that apply.) a. Previous medical history b. Physical appearance c. Age, gender, and culture d. Lifestyle and loss of appetite e. Hair color and style

Answer: a, b, c, d Medical history, physical appearance, age, gender, culture, lifestyle, and loss of appetite should be considered when conducting a pain assessment. Hair color and style are not necessary components of a pain assessment. LO: 36.4

Two adult siblings are caring for their ill mother, who requires 24-hour care. She needs assistance with feeding, bathing, and toileting. One of the siblings takes time to exercise after work, whereas the other goes directly to the mother's home before and after work each day. The nurse recognizes that people may react differently to the same stressors depending on which factors? (Select all that apply.) a. Individual coping skills b. Type of identified stressor c. Amount of perceived stress d. Personal appraisal of the stressor e. Hair color, gender, and skin type

Answer: a, b, c, d The person's coping skills have an impact on how that person perceives and responds to stress. The type and amount of stress as well as the appraisal of stress also affect how the person reacts. Hair color, gender, and skin type are not recognized as factors related to stress reactions. LO: 32.7

A nursing instructor is explaining the teach-back method to nursing students on a medical-surgical unit. The instructor asks the students to identify benefits of using this method of patient education. Which should the students include in their response to their instructor? (Select all that apply.) The teach-back method allows the nurse to determine understanding of information taught and to reteach if necessary. The nurse can rephrase information to the patient if the patient is unable to repeat the information correctly. The nurse can ask the patient to repeat information until it is determined that the patient has verbalized understanding of the information taught. The nurse can teach the patient using pictures, videos, and examples. The teach-back method can be used in any health care setting.

Answer: a, b, c, d, e The teach-back method allows the nurse to determine understanding of information taught and to reteach if necessary by rephrasing the information, using pictures, videos, and examples if the patient is unable to repeat the information correctly. The nurse can ask the patient to repeat information until it is determined that the patient has verbalized understanding of the information taught. The teach-back method can be used in any health care setting. LO: 14.10

Which statements by a patient would indicate the use of effective coping strategies? (Select all that apply.) a. "Each month, my wife and I attend a support group for parents of children with autism." b. "Talking with my spiritual adviser may challenge my thinking on how best to handle this situation." c. "I've invited my son to join me for drinks at the bar each night on his way home from work so that we can spend more time together." d. "We are looking into joining the new health club facility in our neighborhood." e. "After working all day, I eat dinner in front of the television while my family sits at the kitchen table."

Answer: a, b, d Support groups, spiritual advisors, and health clubs all offer services that can enhance coping skills. The daily use of alcohol is not a healthy coping strategy, even if it involves spending time with family. Eating in front of the television promotes obesity and social isolation. LO: 32.7

The nurse is caring for a critically ill patient. What are the contraindications for administering medications by the oral route for this type of patient? (Select all that apply.) a. Vomiting b. Unconsciousness c. Diarrhea d. Penicillin allergy e. Intubation

Answer: a, b, e Nothing that needs to be swallowed should be administered to an unconscious patient due to the risk of aspiration. Medications are unlikely to be absorbed in the patient who is vomiting. An endotracheal tube makes it impossible for a patienT to swallow oral medication. A penicillin allergy affects the type of medication to administer but not the route. A patient with diarrhea may have decreased absorption, but it does not affect the ability to swallow medications safely. LO: 35.4

Which interventions are considered helpful to assist nurses coping with the unexpected death of a patient for whom they cared for many weeks? (Select all that apply). a. Engaging in preferred personal spiritual practices b. Journaling personal reflections surrounding the death of the patient c. Scheduling work to a different shift than the one regularly worked d. Arranging a consultation with the unit manager to discuss a possible unit transfer e. Setting aside time for relaxation activities, such as painting, gardening, or exercising

Answer: a, b, eEngaging in helpful personal spiritual practices, journaling personal reflections, and setting aside time for relaxation activities are all considered helpful strategies for nurses coping with loss and stress. Scheduling to work different shifts and discussing a possible unit transfer are both forms of avoidance that may result in increased stress and delayed grieving. LO: 22.7

A patient has reported a 2-kg (4.4-lb) weight gain over the past 3 days. Which assessment cues should the nurse recognize as additional evidence for a nursing diagnosis of Fluid Volume Overload (Hypervolemia)? (Select all that apply.) Third spacing/edema Potassium intake Bounding, rapid pulse Crackles in lungs e. Dry mucous membranes

Answer: a, c, d An increased hydrostatic pressure or a decreased oncotic capillary pressure can cause fluid to move to the periphery, causing edema or third spacing. This can cause weight gain, as fluid is not removed by the kidneys from the bloodstream. Patients with fluid overload may have a rapid, bounding pulse and/or crackles in the lungs. Potassium intake and dry mucous membranes is not a sign of fluid retention. LO: 39.3

The nurse notices that a patient is becoming short of breath and anxious. Which interventions are independent nursing actions that do not require the order of a primary care provider? (Select all that apply.) a. Elevating the head of the patient's bed b. Administering oxygen by nasal cannula c. Assessing the patient's oxygen saturation d. Evaluating the patient's peripheral circulation

Answer: a, c, dElevating the head of the bed, assessing a patient's oxygen saturation and peripheral circulation are all independent nursing interventions. Before a nurse can legally administer oxygen to a patient, the method of delivery and amount must be ordered by the primary care provider or be part of standing orders for patients experiencing similar symptoms. LO: 8.6

A patient is anxious about upcoming medical treatments and is having difficulty falling asleep. Which nursing interventions would promote the patient's ability to sleep? (Select all that apply.) a. Give the patient a back rub. b. Take the patient for a brisk walk right before bedtime. c. Provide a warm, quiet environment. d. Encourage the patient to eat a large meal in the evening. e. Give the patient a diet carbonated cola beverage. f. Play soft music during the 30 minutes before bedtime.

Answer: a, c, f Giving a back rub; providing a warm, quiet environment; and playing soft music enhance relaxation, which will lead to easier transition into sleep. Brisk exercise, caffeine drinks, and large meals all are contraindicated in the evening because they induce changes that will interfere with sleep. LO: 33.6

A nurse caring for a hospitalized patient with dehydration is told in the shift report that the patient's laboratory results have just come in. The nurse recognizes which abnormal lab values that can reflect a fluid volume deficit? (Select all that apply.) a. Sodium (Na) level 150 mEq/L b. Potassium (K) level 3.5 mEq/L c. Calcium (Ca) level 9.5 mg/dL d. Blood urea nitrogen (BUN) 27 mg/dL

Answer: a, d The sodium and BUN levels are elevated, which can often be seen in dehydrated clients. Normal sodium levels for adults range from 135 to 145 mEq/L. Normal BUN levels for adults range from 10 to 20 mg/dL. The potassium and calcium values are within normal limits. LO: 39.2

A patient has been using herbal medication as part of her daily routine. Which actions should the nurse take? (Select all that apply.) a. Document the herbs as part of the medication history. b. Recommend a reputable company from which to buy herbs. c. Allow the patient to self-administer the herbs with her morning medications. d. Inform the primary care provider of the findings. e. Identify possible adverse effects of the herbal medications.

Answer: a, d, e It is important to include the herbal medications that the patient reports using in the medication history because there is the possibility of interactions among herbals and prescribed medications. The primary care provider should always be informed about the herbs being used by the patient. It is essential for the nurse to identify potential adverse effects of the herbal medications to prevent harmful drug interactions. It is inappropriate for the nurse to recommend a company for the patient to purchase herbal preparations and to allow the patient to self-administer the herbs while in the hospital without a specific physician order. LO: 35.3

When caring for an elderly patient who presents with acute confusion of sudden onset, which test would the nurse expect to be ordered? a. Urine culture and sensitivity testing b. Mini-Mental State Examination (MMSE) c. Swallow evaluation d. Magnetic resonance imaging (MRI) with contrast

Answer: aA major cause of acute confusion in the elderly is infections, including urinary tract infections and pneumonia. Urine culture and sensitivity testing will detect bacteria in the urine and determine proper antibiotic treatment. A MMSE is a valuable tool to assess the progression of dementia. Swallow evaluation is done in patients who are suspected of having a weak or absent gag reflex. MRI with contrast might be done in a patient with confusion after infection has been ruled out. LO: 31.3

Which nursing diagnosis is appropriate if a patient expresses an interest in learning? Ready to Learn Lack of Knowledge Effective Information Processing Health-Seeking Behaviors

Answer: aA patient's expression of an interest in learning would indicate correct use of the nursing diagnosis, Ready to Learn. Lack of Knowledge would indicate the patient has a deficiency of knowledge on a particular subject. Effective Information Processing is the patient's ability to acquire useful information. Health-Seeking Behaviors is active seeking by a person of ways to alter habits to enhance health. LO: 14.8

Which situation indicates the greatest need for collaborative interventions provided by several health care team members? a. Hospice referral b. Physical assessment c. Activities of daily living d. Health history interview

Answer: aHospice referral requires collaboration with many health care team members. Physical assessment and completion of a health history interview are independent nursing actions that can be performed by a nurse alone. Activities of daily living can be completed by patients independently or with the help of a nurse or unlicensed assistive personnel (UAP), requiring little collaboration among health care team members. LO: 8.6

What is the primary difference between a NANDA-I risk nursing diagnosis and a problem-focused nursing diagnosis? a. Related factors are not part of a risk diagnosis. b. There is no cause-and-effect relationship established. c. Defining characteristics are subjective in a risk diagnosis. d. There are no nursing interventions prescribed with a risk diagnosis.

Answer: aRisk diagnoses do not have related factors; problem-focused nursing diagnoses have both related factors and defining characteristics. Risk diagnoses do not establish a cause and effect because they identify potential rather than existing problems. Risk diagnoses identify risk factors (not defining characteristics), subjective or otherwise. Risk diagnoses, like problem-focused diagnoses, have nursing interventions that address a patient's current or potential problem. LO: 7.3

Which statement is an example of the use of situational leadership? The emergency department manager takes a vote on holiday coverage and then responds to a Code Blue by directing orders at the nursing staff. The manager in surgery uses the vacation policy to grant time off and then performs a surgical count in an operating room using a checklist. A vice president of nursing allows the department directors to make a decision about a hospital policy on holiday time and then sides with a nurse who does not want to work the required time. The Chief Executive Officer (CEO) of the hospital instructs the nursing senate to develop a dress code and then changes the dress code after determining he does not like it.

Answer: aSituational leadership adjusts leadership styles to fit the situation. In the correct option, the manager moved from a democratic to an authoritarian leadership style to fit the change in situation. The manager in surgery follows a bureaucratic style by strictly following policy and procedure. The vice president of nursing is following a laissez-faire style because responsibility for the decision is abdicated and the vice president of nursing does not support the policy when needed. The CEO of the hospital is following an authoritarian style of leadership because input of the staff is not valued. LO: 12.2

The nurse supports a patient's decision to decline more cancer treatment and to be cared for by a hospice team, even though the nurse personally thinks the patient should seek more treatment. The nurse is practicing which nursing role? a. Advocacy b. Change agent c. Leader d. Collaborator

Answer: aThe nurse has many roles. In this circumstance, however, the nurse is acting as the patient's advocate. Nurses advocate for the patient and support the patient's decision even when the nurse does not agree with the decision.

What is the best method for the public health nurse to determine whether community members are involved in outdoor physical activity? a. Meet with the parents of high school children. b. Complete a windshield survey of the community. c. Evaluate the number of community health club members. d. Check the local health statistics for the incidence of obesity.

Answer: b A windshield survey will allow the nurse to observe whether people are walking or engaged in physical activity. It also will help the nurse identify single- or multiple-family private and public housing units; health, safety, and social services agency availability; and other essential community attributes. Meeting with parents, evaluating health club membership, or obesity statistics will not assess the concern of the prevalence of outside physical activity.

When should administered medications be documented? At the end of a shift when all medications have been given As given to avoid the possibility of double dosing After every meal to document at least three times daily When the nurse has time before going on break

Answer: b All medications and nursing care should be documented as completed to ensure that documentation occurs in a timely manner. Documentation should occur as soon as possible after assessment, interventions (including medication administration), condition changes, or evaluation. Documentation only at the end of a shift, after meals, or before breaks would not be timely and could lead to medication errors and fragmented care. LO: 10.3

While assessing a patient for domestic violence, the nurse knows that which statement is true regarding domestic violence? a. It is a health risk factor only during young adulthood. b. It occurs across age, socioeconomic levels, and cultural boundaries. c. Young women aged 20 to 24 have the lowest incidence of rape and sexual assaults. d. Women are the only victims of domestic violence.

Answer: b All socioeconomic levels and cultures are affected by domestic violence. It is a health risk for adults of all ages—from young to older adulthood. Young women have the highest rate of rape and sexual assault. Women are not the only victims of domestic violence. Men also may be victims of domestic violence and sexual assaults, although they are less likely to report it or to seek care.

What action should be taken by the nurse first when preparing to administer medications to a patient? a. Check the medication expiration date. b. Check the medication administration record (MAR). c. Call the pharmacy for administration instructions. d. Check the patient's name band.

Answer: b Checking the MAR is the first step in administering medications. Checking the expiration date and administration instructions is done only after the order is verified. Checking the patient's name band is one of the six rights of medication administration, but it is done after the MAR has been checked and the drug, dose, route, and time of administration have been verified. LO: 35.5

If a patient's primary language differs from that of the health care professionals providing care, which action is most appropriate for the nurse to take? a. Use colorful pictures, whiteboards, and gestures to communicate all important information. b. Verify patient understanding of questions asked when the patient responds with continuous affirmative answers. c. Arrange for a professional language translator to be present 24 hours each day. d. Decrease interaction with the patient and family to avoid making them uncomfortable for not understanding.

Answer: b Consistent affirmative answers from a patient in the form of verbal responses or nods may indicate that a patient does not really understand what is being asked and is just trying to be cooperative. It is important for the nurse to double check to make sure a patient understands instructions and questions to ensure safety and proper care. Not all information can be conveyed via pictures or gestures, and a professional interpreter need not be present 24 hours a day. Interpreters can be reached by phone or video conferencing, as needed. Ignoring or avoiding patients or families with culturally diverse backgrounds serves to isolate them and is never appropriate. LO: 21.5

Which description is an example of data? A printout of a patient's history and physical examination A patient's blood pressure and pulse rate The nurse's knowledge of a disease A nurse's interpretation of a change in the patient's condition

Answer: b Data are facts, observations, and measurements, such as blood pressure and pulse rate. A printout of the patient's history and physical is organized information that is meaningful. Knowledge is organized and processed information, such as a nurse's knowledge of a disease. When nurses interpret a change in the patient's condition, they are using wisdom or the use of knowledge and experience to manage and solve problems. LO: 15.1

How best can a nurse evaluate goal attainment for a patient with a culturally diverse background? a. Assume that gender roles will be a challenge to overcome regardless of the patient's ethnicity. b. Base decisions on feedback from the patient and the nurse's professional judgment. c. Collaborate with future community care providers to determine patient strengths. d. Seek input from members of the patient's support system to avoid biased patient responses.

Answer: b Decisions about whether a patient has met treatment goals or outcomes should be based on patient feedback and a nurse's professional judgment. Gender role considerations are unlikely to play a role in evaluation. Future community care providers are unable to help in the evaluation of patient goals before participating in a patient's care. The patient is the primary person from whom information should be obtained in evaluating goals and outcomes.

Which statement by the patient with vertigo lets the nurse know that the patient has understood the home-going instructions? a. "I will buy a visual signal for my smoke detectors." b. "I will have grab bars installed in my bathtub." c. "I will change positions quickly to avoid vertigo." d. "I will get a home phone with amplified sound."

Answer: b Grab bars provide stability for the patient with vertigo. Patients with vertigo should change positions slowly to avoid worsening of the spinning sensation. Visual signals and amplified sound are used in the home of the patient with hearing deficits. LO: 31.6

If a patient is grimacing, what assessment statement or question would be most beneficial in identifying the underlying cause of the nonverbal communication? a. "Did you lose something?" b. "You appear to be having pain." c. "I will turn off the lights and let you rest." d. "May I get you something to relieve your tension?"

Answer: b Grimacing is a common nonverbal sign of pain. Sharing an observation encourages the patient to elaborate on nonverbal communication. Asking the patient whether something is lost indicates that the nurse has not attended to the nonverbal cues of the patient. It is important to do an assessment of the patient before initiating any interventions. LO: 3.2

. Which behavior by the young adult patient indicates an understanding of patient education aimed at reducing the health risks for that age group? a. Smoking only one pack of cigarettes per day b. Limiting alcohol use to an occasional drink c. Using drugs found in a roommate's drawer for anxiety d. Having a relationship with a partner who was threatening in the past

Answer: b Having an occasional drink shows control and moderation. Smoking any amount is damaging to the lungs, and education for cessation is needed. Using any drugs that are not directly prescribed for the person shows a lack of understanding of the risks of drugs. Staying in a relationship that has been dangerous in the past increases the risk of intimate partner violence. LO: 18.4

A patient is seeking information about leading indicators that show the importance of health promotion and illness prevention in the United States. To which government-sponsored resource would the nurse refer the patient for the best comprehensive source of information? a. The American Cancer Society website b. The Healthy People 2030 website c. The Centers for Disease Control and Prevention (CDC) Morbidity and Mortality Weekly Report d. The American Association of Hospitals home page

Answer: b Healthy People 2030 is the most up-to-date site for health indicators in the United States, and it is presented in an easy-to-understand format. The CDC Morbidity and Mortality Weekly Report does not cover all of the indicators, and it may be overly technical for the patient. The same holds true for the American Association of Hospitals home page. The American Cancer Society website would supply the consumer with information specific to cancers as opposed to the comprehensive health information found in the Healthy People 2030 document. LO: 16.3

1. While caring for a patient with advanced multiple sclerosis, the nurse is discussing the difference between hospice and palliative care. Which statement by the patient indicates understanding of the difference between hospice care and palliative care? a. "I will need to get hospice care if I want my symptoms controlled." b. "I can have palliative care right now—even though I am not going to die anytime soon." c. "My doctor has to make the decision if I have hospice care." d. "I can't get any other treatments, even if they are experimental, if I choose palliative care."

Answer: b Hospice care and palliative care are focused on the management of symptoms. Hospice care is provided to those who have a prognosis of less than 6 months to live. Palliative care is provided to any person who needs assistance with management of symptoms. Physicians delineate the prognosis, but the patient and family ultimately make the decision if they want care provided by hospice. LO: 42.7

A client with metastatic cancer shares with the clinic nurse that he has only days or weeks to live. What type of community service would be most appropriate for the nurse to suggest to this client? a. Home health care b. Hospice care c. Forensic care d. Acute care

Answer: b Hospice care provides specialized care for people who are dying. It may take place in a person's home or community facility. Home care is less specialized. Forensic and acute care would not be appropriate for this client. LO: 23.1

What action would be most appropriate for the home care nurse to take if an intrinsic factor appears to be contributing to a client's illness? a. Report the presence of multiple insects in the home to the health department. b. Document the intrinsic factor in the client's electronic health record. c. Explore the possible impact of changing jobs for stress reduction. d. Discuss the danger of having multiple throw rugs with the client.

Answer: b Intrinsic factors include variables such as genetics, age, gender, and ethnic group. These should be documented in the client's electronic health record so that their impact can be taken into consideration in assessing the client's health status. The presence of insects and throw rugs in a client's home and where the client works are extrinsic factors. LO: 23.3

A 55-year-old male is scheduled to have a bowel resection for a diagnosis of colon cancer. He is very nervous about the surgery. How can the nurse help decrease his anxiety? a. Ask him if he is concerned the cancer has spread to other areas of his body. b. Talk to him to find out what is causing his anxiety. c. Question him to find out what the surgeon has told him about the surgery. d. Give him a preoperative medication to help him relax.

Answer: b It is important for the nurse to understand the source of the patient's anxiety. The nurse can then develop a plan to assist him. Asking whether he is concerned that the cancer has spread may increase his anxiety. Questioning him about the surgery does not help the nurse understand the source of his anxiety and may increase it. Giving medication does not help the patient deal with the problem. This may be appropriate after the nurse knows the source of his anxiety. LO: 37.5

In caring for a dying patient, what is an appropriate nursing action to increase family involvement? a. Insisting that all bedside care be performed by the family b. Demonstrating care and supporting family participation c. Expecting the family to consistently perform the patients ADLs d. Refusing all assistance from the family to decrease family stress

Answer: b Many family members would like to be involved in the care of their loved one while the person is dying. It is the responsibility of the nurse to assess the level of involvement in which the family would like to participate related to patient care. Teaching about care measures is a nursing intervention that can be implemented to assist family members during the process of anticipatory grief. Family members should not be expected to meet all of the patient's needs but should not be excluded from caring for their loved one. LO: 42.7

Which action taken by a nurse would reflect application of an appropriate generalization in a patient care setting? a. Assigning same-gender nurses to all patients admitted to the unit b. Asking the dietary intern to verify with Middle Eastern patients whether they eat pork c. Telling the radiology technician that every non-Hispanic family is late for appointments d. Assuming that extended families share financial responsibility for medical bills

Answer: b Middle Eastern people typically do not consume pork products. This generalization would be helpful to use as a baseline when caring for Muslim or Jewish patients. It is always important to ask patients to verify whether they adhere to cultural norms. Same-gender nurses need not be assigned to all patients. Making broad statements regarding people of one culture is stereotyping and hurtful. Assuming anything without asking about patient traditions or preferences is inappropriate. LO: 21.2

If a patient's verbal and nonverbal communications are inconsistent, which form of communication is most likely to convey the true feelings of the patient? a. Written notes b. Facial expressions c. Implied inferences d. Spoken words

Answer: b Nonverbal communication is the more accurate mode of conveying feelings. When a patient's verbal and nonverbal cues are incongruent, it is important to explore observations made by the nurse to discern the true feelings of the patient. Written notes, implied inferences, and spoken words do not provide the opportunity for observing nonverbal cues. LO: 3.2

The nurse is caring for a patient with stated difficulty sleeping after the death of his spouse of 56 years. Which of the following would be the most appropriate action? a. Assess the patient for possible use of sedatives. b. Obtain a health history regarding sleep hygiene. c. Assess the patient's nutritional habits and intake. d. Contact the provider to request a sleep study.

Answer: b Obtaining a health history of the patient's sleep hygiene will help determine interventions that might promote relaxation and sleep. Sedatives are prescribed for only some patients with chronic, ongoing sleep disturbances that interfere with daily life after nonpharmacologic methods have been tried. Although assessing the patient's nutritional habits is an important part of a physical exam, intake is not related to the type of sleep problem described. No symptoms of sleep apnea have been reported; thus, the nurse would not request a sleep study. LO: 33.3

In which scenario is palliative care provided? a. Only in the homes of the terminally ill b. For any chronic illness that requires symptom control c. For cancer patients only in their last weeks of life d. Only in hospital settings based on the seriousness of the illness

Answer: b Palliative care is provided in a variety of settings, including home care, freestanding inpatient units, hospitals, long-term care facilities, and prisons. It is also administered to the homeless and to patients with any disease or illness that has been determined to be chronic and in need of symptom control. Any patient who is experiencing symptoms—physical, psychological, or spiritual—benefits from palliative care. Once a patient is terminal or has less than 6 months to live, the patient can choose to seek hospice care. LO: 42.6

A 75-year-old patient with chronic obstructive pulmonary disease (COPD) gets arterial blood gases ordered. What is the nurse's interpretation of the arterial blood gas results (pH 7.33, PCO2 58, PO2 83, HCO3− 33)? Partially compensated metabolic alkalosis Partially compensated respiratory acidosis Uncompensated metabolic acidosis Uncompensated respiratory alkalosis

Answer: b Patients with COPD tend to have chronic carbon dioxide retention. The patient is slightly acidotic (i.e., arterial pH below 7.35) with a higher than normal partial pressure of carbon dioxide (PCO2), which is inverse and therefore respiratory in nature. The compensatory response to respiratory acidosis is buffering, as indicated by the higher than normal bicarbonate (HCO3−) level. The increase in bicarbonate only partially shifts the pH toward normal, but partial compensation prevents the acid-base imbalance from becoming life-threatening. The kidneys will continue to compensate in an attempt to bring the pH into the normal range. LO: 39.2

The nurse has been assigned the same patients for the past 4 days. Two of the patients demand a great deal of attention, and the nurse feels anxious and angry about being given this assignment again. What action would demonstrate the most effective way for the nurse to cope with the patient care assignment? a. Share complaints about the assignment with the nurse manager. b. Prioritize the patients' needs and identify a specific time period to care for each patient. c. Talk with the patients and explain that they cannot expect so much personal attention. d. Trade assignments with another nurse who is unaware of the concerns regarding the patient assignment.

Answer: b Prioritizing care and setting aside time to spend with specific patients constitute the most effective coping strategy for the nurse to use. Sharing concerns with the nurse manager does not demonstrate strong problem-solving skills; it merely shows a desire to complain. Trading assignments diminishes the continuity of care during which patient trust and nurse-patient relationships are developed. Patients are always the focus of nursing care and should not be given the impression that the nurse does not have time to care or listen to their concerns. LO: 32.8

The nurse is performing a health assessment on a 15-year-old female patient. Which is the best way to obtain accurate information regarding her sexual activity? a. Ask the mother about the girl's sexual activity. b. Privately ask the girl about her sexual activity. c. Warn the girl about the dangers of sexual activity. d. Ask the girl if she wants birth control.

Answer: b Privately asking the girl about her sexual activity in an open and nonjudgmental fashion is the best way to obtain accurate information. Asking the mother about the girl's sexual activity may be embarrassing for the teen and her mother and may not provide accurate information. Warning the girl about the dangers of sexual activity is making a judgment. It is important to provide a nonjudgmental approach with teens. The teen may have additional questions about sexuality beyond the topic of birth control; thus, it is important to approach the teen in a manner that encourages open discussion. LO: 17.8

A patient is newly diagnosed with diabetes and requires insulin injections. He requests information about classes offered by the diabetes educator. Which type of coping technique is this patient using? a. Emotion-focused b. Problem-focused c. Avoidance d. Denial

Answer: b Problem-focused coping techniques are aimed at altering or removing a stressor. If this patient gains the skills to administer his own injections from the diabetes educator, he will remove a major stressor associated with a new diabetes diagnosis. Emotion-focused coping techniques, avoidance, and denial are all psychological techniques, rather than a psychomotor-based activity, which is required in this situation. LO: 32.1

When a patient is beginning a regimen of an antidepressant medication, which information should the nurse include in the medication teaching as it pertains to sexuality? a. "Your partner will be pleased because your sexual functioning is going to improve." b. "You may find that your desire for sex will decrease while on this medication." c. "Your skin will probably become supersensitive to touch, so you may need to change your activity during sex." d. "You will be unable to have an erection while taking your antidepressants."

Answer: b Reduced sexual desire can be a side effect of antidepressant use. Skin sensitivity and erectile dysfunction are not side effects. Improved sexual function is not a specific effect of antidepressant administration. LO: 24.8

A 2-year-old child insists on having a drink of water and having a story read to him and says, "Good night, sleep tight" at bedtime every night. The nurse knows the child is exhibiting which type of behavior? a. Controlling b. Ritualism c. Obsession d. Compulsion

Answer: b Ritualism offers the toddler a sense of security and comfort. The child is not trying to control his behavior with the ritual. Obsession and compulsion are terms commonly used in patients in older age groups and describe maladaptive behaviors. LO: 17.5

An active older patient has been frequently evaluated for minor problems at the clinic since the death of her husband 3 months earlier. During one of her visits, she states that she is tired from not sleeping well, feels restless, and is nervous about attending her Tuesday night bridge club. Which type of holistic health model intervention should the nurse employ to help the patient cope with the loss of her husband? a. Encouraging prn use of antianxiety medication ordered by her provider b. Sharing the value of music therapy to address anxiety about her loss c. Explaining that she will be over the loss of her husband in a few months d. Encouraging a gradual reentry into social interaction and activities with friends

Answer: b Sharing the value of using music therapy to reduce anxiety surrounding her loss is the most holistic health model intervention listed. This type of distraction is a holistic technique that works well in the form of music, painting, and expressive dance. Instructing the patient to use drugs is not part of the holistic approach. The patient may not be ready to re-engage in a full activity schedule, and it is unlikely that her feelings of loss will ever totally resolve. LO: 16.2

The nurse decides to access a systematic review database to determine EBP related to the patient's treatment plan for a diagnosis of otitis media (i.e., ear infection). What database can provide that type of resource? a. Cumulative Index of Nursing and Allied Health Literature (CINAHL) b. Cochrane c. PubMed d. MD Consult

Answer: b The Cochrane database provides systematic reviews of the literature. CINAHL and PubMed provide primary research articles, and MD Consult provides background information. LO: 13.5

In using the Ex-PLISSIT model, what is the first action initiated by the nurse? a. Present basic information about sexual functioning. b. Ask permission to begin the sexual assessment. c. Inquire about any medications the patient is taking. d. Ask the patient about sexual activity and practices.

Answer: b The Ex-PLISSIT model is an expansion of the PLISSIT model, in which permission-giving questions are required at each level. These questions allow the clinician to check in with the person, clarify the person's situation and experience, and provide the clinician with an opportunity to reflect on the interventions. LO: 24.10

What is the best way for a nurse to avoid crossing professional practice boundaries with patients? a. Spend extensive time with a patient without visitors b. Focus on the needs of patients and their families c. Intervene in problematic patient relationships d. Relay personal stories when unsolicited

Answer: b The best way to avoid crossing professional practice boundaries is to focus on the patient's needs and those of the patient's family. Nurses should consistently avoid unsolicited self-disclosure of personal information, intervening in patient relationships, and spending excessive amounts of time with one patient. In addition to keeping patients' secrets, gaining personally from a relationship with a patient and engaging in sexual overtures or sexual behavior with a patient are considered violations of professional boundaries. LO: 11.8

When teaching female reproduction to a group of high school students, the nurse uses what term to indicate the cessation of a woman's menstrual activity? a. Menarche b. Menopause c. Premenstrual syndrome d. Menstrual dysfunction

Answer: b The definition of menopause is the cessation of a female's menstrual activity. Menarche is the onset of menstruation. Premenstrual syndrome is a set of specific symptoms that occur before the monthly menstrual cycle. Menstrual dysfunction refers to altered patterns of menstruation associated with various disorders. LO: 24.1

The hospital has implemented a new electronic medication administration record (eMAR). What is true about the use of this new tool? Verifies medication dosages Reduces medication administration errors Eliminates the need to count narcotics Requires a hard copy of the MAR to be printed

Answer: b The electronic medication administration record (eMAR) reduces medication errors by requiring the nurse to scan the patient's identification band and the medication. Although the eMAR alerts the nurse to potential errors such as the wrong dose, it is the nurse's responsibility to verify all information before administration of a medication. Narcotic counts are still kept in the electronic system. A hard copy of the MAR is not necessary. LO: 15.3

A nurse in the emergency department is caring for an adult patient with traumatic abdominal injuries. The patient's pulse rate has increased from 90 to 120 beats/min over the past hour and the patient is experiencing orthostatic hypotension. For which imbalance should the nurse assess? Respiratory acidosis Extracellular fluid volume deficit Metabolic alkalosis Intracellular fluid volume excess

Answer: b The elevated heart rate and orthostatic hypotension show extracellular fluid volume deficit. With the client's injuries, there is the potential for internal bleeding. As circulating blood volume decreases, the heart rate increases to maintain normal cardiac output, and the patient may experience orthostatic hypotension and lightheadedness with position changes. Respiratory acidosis and metabolic alkalosis do not have as a symptom a rapidly increasing pulse rate. Intracellular fluid volume excess causes pulmonary congestion and cerebral edema. LO: 39.3

What is the role of the nurse in securing an informed consent for a surgical procedure? a. Ensuring that the patient signs the informed consent document b. Signing the consent form as a witness c. Ensuring that the patient does not refuse treatment d. Refusing to participate based on legal guidelines

Answer: b The nurse is acting as the witness. The physician has the primary responsibility of securing the signed consent form. The nurse serves as an advocate for the patient. It is unethical to force patients to have a treatment that they do not want. Legal guidelines allow nurses to witness the patient signing a consent form. LO: 37.8

A nurse has performed a physical examination of the patient and reviewed the laboratory results and diagnostics on the patient's chart. The nurse is performing which specific nursing function? a. Diagnosis b. Assessment c. Education d. Advocacy

Answer: b The nurse is performing the first step in the nursing process—assessment.

The nurse enters a patient's room and notices that the patient has not been out of bed since the previous day. The patient states that his condition has made him bedridden, although the nurse knows that he is capable of independent ambulation. Which type of reaction is the patient exhibiting? a. Ambivalence to symptoms b. Illness behavior c. Diminished functional ability d. Overreaction to illness

Answer: b The patient is defining and interpreting his disease symptoms according to his beliefs about illness and how to respond to it. The patient's ability to ambulate is intact. Overreaction to illness is a subjective anomaly, and the patient is not ambivalent about his diagnosis. LO: 16.5

As the nurse explained the preoperative instructions to the patient, the patient's older brother suddenly stepped into the doorway and yelled, "People who go under the knife always die. Don't do it! They're going to kill you." What type of higher-order belief is the patient's older brother displaying? a. Distress b. Stereotype c. Prejudice d. Denial

Answer: b The patient's brother is verbalizing a stereotype, which is a belief about a person, group, or an event that is thought to be typical of all others in that group. Although it is true that people occasionally die during surgery, it does not always happen as the brother fears. Distress is incorrect; the male is distressed, but distress is not a higher-order belief. Prejudice is incorrect, because a prejudicial belief is a preformed opinion, usually an unfavorable one, about an entire group of people based on insufficient knowledge. Denial is wrong, because he is not in denial, which is defined as a behavior of refusing to admit something is true. LO: 2.2

What specific aspect of a profession does the development of theories provide? a. Altruism b. Body of knowledge c. Autonomy d. Accountability

Answer: b Theories establish a specific nursing body of knowledge that is unique to the discipline, which is one criterion of a profession.

Knowledge gained from research in the 1970s about placing infants on their backs to prevent sudden infant death syndrome (SIDS) was not recommended to parents until the 1990s. This is an example of what barrier to EBP? a. Proliferation of research b. Implementation delay c. Information needs not being met d. Lack of readily available resources

Answer: b There is an implementation delay of approximately 17 years from clinical research to integration into practice. Proliferation of research is the large amount of research that is conducted. Lack of resources refers to a reduced number of databases that provide the information being sought, and information needs not being met refer to the inability to readily find the answers to questions. LO: 13.4

A hospital has implemented the use of electronic health records (EHRs). While learning to use this system, the nurse realizes that EHRs may do which of the following? Limit access to the patient record to one person at a time Improve access to patient information at the point of care Negate the use of nursing documentation Increase the potential for medication errors

Answer: b Use of EHRs can improve access to patients' information. An unlimited number of people at a time can access a patient's medical record. Nursing documentation is an essential part of nursing care. The potential for medication errors should decrease when electronic medication administration records are used properly. LO: 10.2

A patient's sister comes to visit and asks to read the patient's medical records. What is the best response by the nurse? Settle her in a chair at the nurses' station and give her access. Respond that the contents of a patient's medical records are private and confidential. Tell her she can read the medical records only if the patient sits with her. Distract the sister by changing the subject and then walking away.

Answer: b Without special permission from the patient, only those with a need-to-know-the-information-for-care reasons have access to the medical record. The patient has a legal right to control access to personal information, and the nurse should not give the sister access, even with the patient present. It is best to be honest and explain the patient's legal rights rather than avoiding the subject. LO: 10.4

Which defense mechanism is being exhibited when a 27-year-old patient insists on having a parent present during routine care? a. Denial b. Regression c. Repression d. Displacement

Answer: b Young adults who require their parents' presence for routine care are exhibiting regression, which is behavior consistent with earlier stages of development. Patients in denial refuse to see the reality of their situation. Repression is storing painful feelings in the unconscious, causing them to be temporarily forgotten. Displacement transfers emotional energy away from the actual source of stress to an unrelated object or person. LO: 3.8

A patient who has type 2 diabetes is admitted to the hospital with a fractured left femur. What are the two highest-priority actions for the nurse to implement? a. Starting an intravenous solution of lactated Ringer solution b. Administering ordered pain medication after taking vital signs c. Glucometer testing to monitor the patient's blood glucose level d. Giving the ordered prophylactic antibiotic to prevent infection e. Encouraging assisted ambulation to avoid blood clot formation

Answer: b, c The highest-priority actions are to administer the patient's prescribed pain medication and checking the patient's blood glucose level. Pain control and blood glucose monitoring are very important as stress caused by pain can cause an increase in blood glucose. Starting an ordered IV solution and administering prophylactic antibiotics can follow analgesia administration and glucometer testing. Encouraging assisted ambulation is not indicated at this time. A sequential compression device (SCD) may be ordered for the patient's right, uninjured leg to prevent blood clot formation before the patient has surgery to repair the fractured femur.

For a patient with a nursing diagnosis of Dehydration, the nurse recognizes which cues as signs and symptoms of dehydration? (Select all that apply). Hypertension Elevated urine specific gravity Dry mucous membranes Weak, thready pulse Pale yellow urine

Answer: b, c, d Depending on the severity of fluid volume deficit, the patient may have hypotension. Hypertension occurs with fluid volume overload. With dehydration, the urine becomes concentrated with an elevated specific gravity and usually turns dark amber colored (not pale yellow). The skin is usually flushed and dry, and the pulse is weak and thready. LO: 39.4

The nurse has just begun an infusion of packed red blood cells (PRBCs). Which of the following cues should the nurse recognize as indicating a transfusion reaction that warrants stopping the infusion? (Select all that apply.) Patient complains of weakness and fatigue. Patient complains of feeling itchy. Patient is shivering and complains of chills. Temperature increased from 99.1° degrees to 101.3° F. e. Patient complains of nausea.

Answer: b, c, d Weakness and fatigue are commonly experienced in anemic patients needing a blood transfusion. Itching or hives can be a sign of an allergic reaction. Shivering/chills can indicate hemolytic or nonhemolytic reactions as well as infection due to bacterial contamination. An increased temperature of more than 2 degrees Fahrenheit indicates a reaction; the infusion should be stopped and primary care provider and blood bank notified. Nausea is not a side of effect of a transfusion reaction. LO: 39.6

The nurse is caring for a 6-year-old patient in the emergency department who just had a full left leg cast placed for a fracture. As the nurse is reviewing the discharge instructions with the patient's mother, she states, "You don't have to go over those—I'll read them at home." What should the nurse do? Contact the physician immediately. Consider the possibility of health literacy limitations and assess further. Stop the teaching, because the mother obviously has taken care of casts before. Explain to the mother that reading the instructions with her is required.

Answer: bA patient's mother may have limited reading skills or health literacy and should be further assessed. Contacting the physician in this situation would not be appropriate because ensuring that the patient and family understand discharge instructions is the responsibility of the nurse. Assuming that the mother has taken care of casts in the past may be inaccurate. Stating that reading the instructions with the nurse is a requirement does not ensure that the patient or mother comprehends the instructions. LO: 14.7

Which nursing goal is written correctly for a patient with the nursing diagnosis of Risk for Infection after abdominal surgery? a. Nurse will encourage use of sterile technique during each dressing change. b. Patient's white blood count will remain within normal range throughout hospitalization. c. Patient's visitors will be instructed in proper handwashing before direct interaction with patient. d. Patient will understand the importance of cleaning around the incision with a clean cloth during bathing.

Answer: bA patient's white blood cell count is a laboratory test that is a measurable indicator of infection. The correct answer is also patient focused and realistic. Encouraging the use of sterile technique by the nurse during each dressing change and instructing the patient's visitors in the proper handwashing technique before direct interaction with the patient are not patient focused. The patient understanding the importance of cleaning around the incision with a clean cloth during bathing uses a non-measurable verb, which should be avoided when formulating patient goals. LO: 8.3

A 57-year-old male patient who was hospitalized with an admitting blood pressure of 242/116 asked the nurse if his family could bring in some meat and vegetable dishes from home. He explained that he cannot eat the foods on the hospital menu, because it is summer and the hospital is offering only chicken and fish, which in his culture are "hot" foods that will interfere with his healing. Which response by the nurse would best demonstrate an application of Leininger's theory? a. Discourage the family from bringing in food, explaining that the idea of "hot" and "cold" foods is a superstition without scientific basis. b. Negotiate home-prepared food options with the patient and his family to ensure that treatment for the patient's blood pressure is supported. c. Explain that the patient will need to have home-prepared foods evaluated by the dietary staff to ensure that they are acceptable options. d. Tel

Answer: bAccording to Leininger's theory, negotiation and adaptation are part of what nurses do to accommodate the patient's cultural ways of life. If the foods from home have low concentrations of sodium or other ingredients that are known to negatively impact blood pressure, the nurse can accommodate the patient's beliefs and cultural dietary practices as well as the medical plan of care. Rejecting the patient's cultural traditions and/or accepting them without regard for the well-being of the patient are unacceptable actions. Food given to patients from family members does not need to be evaluated by the dietary staff before consumption. LO: 2.5

If a patient is exhibiting signs and symptoms of each of these nursing diagnoses, which should the nurse address first while planning care? a. Fatigue b. Acute Pain c. Lack of Knowledge d. Disturbed Body Image

Answer: bAcute Pain is the most urgent nursing diagnosis of these options to address. Fatigue may be a result of the pain and may be alleviated if the patient's pain level is reduced. Disturbed Body Image and Lack of Knowledge can be treated only after the patient's pain level is at an acceptable level. Both of the last two diagnoses require teaching, during which the patient needs to concentrate. The ability to concentrate is affected by pain level. LO: 8.2

A 58-year-old man is admitted for a small-bowel obstruction late Saturday night. The admitting orders include the need to place a nasogastric (NG) tube to low intermittent suction. During the assessment, the nurse determines that the patient does not speak English. Which action should the nurse take first before placing the NG tube? Use two additional staff members when placing the tube so the patient can be restrained if needed. Request an interpreter per facility protocol. Do not place the NG tube because the physician would not want to frighten the patient. Document the inability to place the NG tube due to lack of ability to communicate.

Answer: bAn interpreter employed by the hospital would be the best choice so that someone in the room can communicate and provide comfort for the patient. Taking additional staff into the room may increase the patient's anxiety, thereby decreasing his ability to comprehend the instructions. Although the physician would not want to frighten the patient, the physician ordered the nasogastric (NG) tube for the benefit of the patient; therefore, it needs to be placed. Documenting the inability to place the NG tube due to lack of means of communication is not acceptable and does not ensure that the patient gets the needed treatment. LO: 14.6

A patient is faced with a life-threatening diagnosis and is struggling to make decisions based on his/her belief system. The nurse listens, then offers some ideas about how values and beliefs can impact decision-making. The patient says, "I've already heard all of that before, and I don't agree with any of it." How should the nurse proceed? a. Ask for an explanation of the patient's values. b. Ask the patient to share what the patient believes. c. Ask the patient about the patient's prejudicial attitude. d. Confront the patient about the apparent values conflict.

Answer: bBy asking the patient to share what he or she believes, the nurse is asking an open-ended question to determine what part of what the nurse is saying the patient believes and what part the patient does not believe. Asking the patient to explain his or her values is incorrect, because there is no mention in the stem about the patient saying his or her values are different from what the nurse is trying to say. Asking the patient about his or her prejudicial attitude is incorrect. There is nothing in the stem that indicates a prejudicial attitude. Confronting the patient about the values conflict that the patient is experiencing is incorrect, because there is nothing in the stem that indicates that the patient is experiencing a values conflict. The patient simply does not want to hear the ideas the nurse tried to share.

A nurse is working a night shift after several months of working day shift. What action does the nurse take to protect patient safety? a. Take a meal break at midnight. b. Plan critical tasks for early in the shift. c. Ask another nurse to administer all medications. d. Turn up lights on the unit to maintain alertness.

Answer: bCritical tasks should be performed early in the shift before the nurse becomes fatigued. The 4 a.m. window is when most people become the sleepiest during a night shift. Thus, it is important that noncritical tasks be planned for this time and that extra care be taken with patient care tasks. A meal break at midnight may be too early to prevent hunger for the entire shift and is not directly related to patient safety. It is not necessary to have another nurse administer all medications if the nurse is aware of the high-risk time for care tasks. Increasing the amount of light is likely to impair the sleep of all patients on the unit. LO: 33.6

A nurse manager is trying to improve patient satisfaction ratings for her area of responsibility. The manager meets with the staff and forms an ad hoc committee to address the issues around the problem. This is an example of what style of leadership? Bureaucratic Democratic Laissez-faire Autocratic

Answer: bThe nurse manager fits the description of the democratic style because the staff members have input into the solution of the problem. A bureaucratic manager develops policies and procedures to follow or reinforces existing policies and procedures. A laissez-faire manager presents the problem to the employees but, rather than offering a plan for addressing it, this type of manager asks employees to solve the problem on their own. An autocratic manager uses the threat of punishment or promise of rewards to solve the problem. LO: 12.2

A nurse is volunteering in the community to educate parents to increase the number of children in the school district who are immunized. The nurse oversees the activities of a group of volunteers. Which role best describes the nurse's activity in this situation? Management Leadership Volunteerism Activism

Answer: bThe nurse meets the criteria for leadership because the nurse is acting in an informal role, not as part of a managerial structure in an organization. Volunteerism and activism are not defined as management or leadership functions. LO: 12.2

The nurse in the emergency department is caring for an 8-year-old who has had a serious asthma attack. When the nurse attempts to explain the problem to the child's mother, she smells cigarette smoke on the mother's breath. The nurse asks the mother if she has been smoking, and the mother responds, "Yes, and I know they've told me before I can't smoke around him." What should the nurse do next? a. Ask the patient's mother what she values more, her child or her habit. b. Ask the patient's mother to explain what she believes about smoking and asthma. c. Ask the patient's mother about her prejudicial attitude toward smoking. d. Confront the patient's mother about the values conflict she's experiencing.

Answer: bThe nurse should begin by asking the mother what she believes because the nurse does not know at this point. When working with a patient who has an addiction, the nurse should begin at the assessment phase of the nursing process and attempt to build a trusting relationship with the patient. Asking the mother what she values more, her child or her habit, is incorrect, because the issue is not about the mother's values but about what she knows and what she believes. Asking the mother about her prejudicial attitude toward smoking is incorrect, because there is nothing in the stem to indicate that the mother is prejudiced toward or against smoking. Confronting the mother about the values conflict she is experiencing is incorrect, because there is nothing in the question to indicate that the mother is having a values conflict. She may not believe what the health care professionals are telling her or she may not believe that she can quit smoking. She may need to be convinced that she can do it, and the best way to make that happen is to build a trusting relationship with her rather than alienate her with accusatory remarks. LO: 2.3

A patient has just experienced a cardiac arrest on the unit. The nurse has implemented the acute care plan for management of code situations. What is the next step the nurse should take? a. Resume all interventions for previously identified nursing diagnoses. b. Perform the steps of the nursing process related to the patient's current condition. c. Seek physician input related to updating the nursing diagnosis statements. d. Evaluate the success of the acute care plan for management of the cardiac arrest.

Answer: bThe patient's condition requires immediate performance of the lifesaving steps of the nursing process. All other answers are secondary actions. The nurse later resumes all interventions for previously identified nursing diagnoses and evaluates the success of the acute care plan for management of the cardiac arrest. Nurses do not seek the input of the physician for creation of nursing diagnoses. LO: 7.5

Which nursing intervention is most important to complete before giving medication to a patient? a. Provide water to aid in the patient's ability to swallow the medication. b. Double-check the patient's allergies before giving the drug. c. Ask the patient to verify having taken the medication before. d. Place the patient in a side-lying position to prevent aspiration.

Answer: bVerifying patient allergies before administering medication is the most important intervention listed to ensure patient safety. Providing water may or may not be necessary depending on the type of medication being administered. Although it is okay to ask a patient about having taken a medication previously, it is not routinely done or most important. It is preferable to have patients sit up while taking medication unless contraindicated. LO: 9.5

Which ICNP nursing diagnosis is most appropriate for a young immigrant who expresses concern for the safety of his family members who were unable to relocate with him out of a war zone? a. Risk for Spiritual Distress b. Impaired Role Performance c. Impaired Family Process d. Difficulty Coping

Answer: c A key factor in Impaired Family Process is a situational crisis that causes a change in communication and emotional and mutual support, all of which are present in this case. None of the information provided indicates a spiritual crisis, ineffectiveness of coping, or impairment of role performance. LO: 21.5

Which nursing intervention is the best example of patient advocacy? a. Collecting blood samples according to the physician's order each morning b. Assessing the vital signs of a patient who is receiving a blood transfusion c. Seeking an additional analgesic medication order for a patient who is experiencing severe pain d. Accompanying an ambulating patient who is walking for the first time after undergoing surgery

Answer: c Advocacy requires a nurse to work on behalf of others who may be unable to speak for themselves. When a patient is in pain and the physician or primary care provider is not present, a nurse must advocate for the patient's needs by initiating contact with the person responsible for addressing an immediate need. In this case, an order for additional pain medication is needed, which requires collaboration with the patient's physician. Collecting blood samples, assessing vital signs, and assisting a patient with ambulation are primary responsibilities of the nurse that do not require advocacy to meet the patient's need. LO: 11.2

. The nurse administered intravenous morphine at 0830. At what time will the nurse evaluate the patient for pain relief? a. 1000 b. 1030 c. 0900 d. 0930

Answer: c After administering intravenous medication, check the patient in 15 to 30 minutes for relief from pain. Intravenous medication is injected directly into the bloodstream and bypasses the gastric system metabolism. LO: 36.8

A visually impaired diabetic patient states that he has lost the call light. What is the next action the nurse should take? a. Clip the call light closer to the patient. b. Tell the patient that the call light is clipped to the bed. c. Describe the call light location; then, take the patient's hand and guide it to that location. d. Instruct the patient to verbally call for a staff member because "someone is always nearby."

Answer: c Always leave the call light within easy reach of the patient. Use of the patient's senses of touch and hearing enables the patient to locate the call light easier. Simply telling the patient that the call light is clipped to the bed is not adequate because the patient will not know where on the bed to look. Verbally calling for the nurse is not acceptable because the nurse and other staff members might be out of hearing range. LO: 31.6

A patient requests a copy of his medical record. What is the correct response by the nurse? Inform him that his record is the property of the facility and cannot be accessed by anyone but staff. Tell him that the Code for Nurses does not allow you to give him access to his records. Acknowledge that he has the right to have a copy of his records, and make arrangements per facility policy. Refer his request to the hospital administrator because all such requests need to go through proper channels.

Answer: c As part of the Health Insurance Portability and Accountability Act (HIPAA) of 1996 and updated in 2009 in The American Recovery and Reinvestment Act (ARRA), patients' rights include obtaining, viewing, or updating a copy of their own medical records. Usually, an electronic health record (EHR) copy is sent to the patient within 30 days. Facilities can charge the patient for the cost incurred in copying and sending medical records. Methods for implementation vary by facility and type of medical record. The Code for Nurses does not control who has access to medical records. Requests would go through the medical records department or the person or department responsible for obtaining and copying patient records. LO: 10.4

What would be the best therapeutic response to a patient who expresses indecision about recommended chemotherapy treatments? a. "Can you tell me why you are undecided?" b. "It's always a good idea to have chemotherapy." c. "What are you thinking about the treatments at this point?" d. "You should follow whatever your health care provider recommends."

Answer: c Asking open-ended questions allows patients to share freely on a subject. "Why" questions, using closed-ended questions, and giving advice are all nontherapeutic communication techniques that limit patient reflection and sharing on topics of concern. LO: 3.7

Which statement best describes the dose of prescribed pain medication that a nurse should administer given pharmacologic treatment considerations? a. The smallest dose possible to avoid opioid addiction b. The smallest dose possible to decrease adverse effects c. A dose that best manages pain with fewest side effects d. A large dose initially to decrease the initial level of pain

Answer: c Based on the patient's report of pain, the nurse administers the dose of medication that is effective in relieving pain without causing adverse side effects. Administering too small of a dose does not relieve pain. Administering a large dose may result in unwanted side effects. Addiction to narcotics is rare. LO: 36.8

Which intervention should the college health clinic nurse implement as a secondary prevention strategy to identify students at risk for diabetes? a. Nutrition education on high-protein food availability b. Promotion of registration in fitness classes c. Blood glucose screening at the health fair d. Administration of prescribed insulin

Answer: c Blood glucose screening is the only secondary prevention strategy listed. Education and promotion of fitness classes are primary prevention strategies, and administering insulin is a tertiary prevention intervention. LO: 23.2

4. The grandmother of two children, 8 and 10 years of age, has died. Their mother asks the nurse what she should do about her children attending the funeral. What is the nurse's best response? a. "Take them to the funeral—they need closure. Many children attend funerals in today's society." b. "Do not take them to the funeral—they are too young to be exposed to the emotions that are demonstrated at funerals." c. "Talk to your children about how they feel about attending the funeral and encourage them to ask questions and talk about their concerns." d. "Talk to your children about what your mother meant to you and how much she cared for them, and then see if they really want to attend the funeral."

Answer: c Caregivers should be encouraged to openly and honestly answer any questions the child may have as they are evaluating the child's responses to the loss while determining whether the child should attend the funeral of a family member. If young children are going to attend the funeral, they should be prepared for what they will see, who will be there, what they may feel, how they may see other people grieving, and what they will be doing during the time that they are at the funeral. It is essential to explain to the child what the body will look like and the fact that the deceased will not talk, move, or breathe. Children should be allowed to attend funerals based on their own abilities to understand the loss, but they should not be forced to attend if they are fearful or have a strong negative reaction to the loss. The nurse should not give her opinion about the children attending or not attending the funeral but can encourage the mother to evaluate each child's feelings about the loss. The parents should make the decision about the children attending the funeral, not the children or the nurse. LO: 42.7

. What is the most important action to implement when providing spiritual care in nursing practice? a. Contacting the health care facility chaplain b. Completing the FICA spiritual assessment and referring the patient, as needed c. Recognizing situations and patient behaviors indicating a spiritual need d. Spending some time in self-reflection

Answer: c Changes in spiritual needs happen in the moment, and it is critical for nurses to recognize when a spiritual need arises. The FICA tool is not sensitive to daily changes in spiritual needs. Calling a chaplain may be an appropriate intervention when the need arises. Nurses should integrate self-reflection in their own spiritual practices to find meaning in their life experiences, but that is not the most important aspect of providing spiritual care. LO: 22.4

Making prejudicial, untrue statements about another person during conversation may expose a nurse to being charged with what offense? a. Libel b. Assault c. Slander d. Malpractice

Answer: c Conversation that includes prejudicial and false statements about another person is an example of oral defamation of character or slander. Libel is the written form of defamation of character. Assault is a threat of bodily harm accompanied by a sense of imminent danger. Malpractice is professional negligence caused by unsafe practice. LO: 11.7

In the immediate postoperative period after open-heart surgery, a patient who is not a diabetic has elevated blood glucose levels. What physiologic stress response would the nurse recognize as being directly responsible for the patient's increased blood sugar? a. Release of epinephrine b. Circulation of endorphins c. Increase in corticosteroids d. Secretion of corticotropin-releasing hormone (CRH)

Answer: c Corticosteroids increase serum glucose levels and inhibit the inflammatory response. Patients who have experienced extreme physiologic stress will often require short-term insulin therapy until their corticosteroid and blood glucose levels return to normal. Epinephrine, CRH, and endorphins all respond to stress; however, corticosteroids are directly responsible for the increase in this patient's blood sugar. LO: 32.2

The nurse is caring for a patient with hypocalcemia who does not like milk. Which food should the nurse encourage the patient to consume? Cod Eggs Spinach Tomatoes

Answer: c Dark leafy vegetables such as spinach, kale, turnip greens, broccoli, Brussels sprouts, and cabbage are sources high in calcium. LO: 39.1

Which term indicates a mental health disorder that is frequently seen in older adults? a. Schizophrenia b. Bipolar disorder c. Depression d. Posttraumatic stress disorder (PTSD)

Answer: c Depression is one of the psychiatric illnesses appearing frequently in the older adult population. Schizophrenia typically is seen in younger adult populations. Bipolar disorder is usually identified before age 65 years. PTSD can be seen in any age group and usually is related to previous experience of a traumatic event. LO: 18.7

The nurse has been caring for a 65-year-old male patient who has just died. In planning for follow-up bereavement care, the nurse knows that which person is at risk for disenfranchised grief? a. A daughter who lives in a different state b. The son who was with the patient when he died c. An estranged ex-wife of the patient who lives nearby d. The 16-year-old grandchild of the patient

Answer: c Disenfranchised grief, a term coined by Kenneth Doka, may occur with any loss that is not validated or recognized. This type of grief is encountered when a loss is experienced that cannot be openly acknowledged or publicly shared by the grieving person. An ex-wife who has been estranged from the deceased may not be able to openly express the grief that she may feel over the loss of someone who once played a significant part in her life. Other family members (such as a daughter who lives in another state, a son who has been active in the patient's care, or a grandchild) are able to openly express their grief and are viewed by society as having an acceptable grief response. LO: 42.2

. For which person seen at the primary care provider's (PCP's) office appointment would patient and family education be most critical? a. A 24-year-old male patient with a cold virus and on no medications b. A 45-year-old male patient on metformin for type 2 diabetes for the past 3 years c. A 75-year-old female patient just prescribed the anticoagulant warfarin d. A 40-year-old male asthmatic patient diagnosed 10 years ago and on albuterol

Answer: c Educating the patient and family members to the side effects of the anticoagulant warfarin and its potential for bleeding is a critical part of care for the 75-year-old female patient who has just started taking this medication. A 25-year-old male patient is young, and his immune system has the potential to fight off a cold virus. Education about a cold versus the flu would be helpful but is not critical. A middle adult who has had diabetes and has been maintained on this medication for 3 years would have had education when he was first started on the medication. A patient who has been using asthmatic medication for the past 10 years is likely to be knowledgeable about the medication from previous use.

Which statement best serves as a guide for nurses seeking to learn more about ethnicity? a. Ethnicity, like culture, generally is based on genetics. b. A patient's ethnic background is determined by skin color. c. Ethnicity is based on cultural similarities and differences in a society. d. Culture and socialization are unrelated to the concept of ethnic origin

Answer: c Ethnicity is based on cultural similarities and differences in a society or nation. The similarities are with members of the same ethnic group; the differences are between that group and others. Ethnicity is not based on or determined by genetics or skin color. Culture, ethnicity, and socialization are all related concepts. LO: 21.1

An 8-year-old girl is newly diagnosed with type 1 diabetes. The nurse may expect fear and crying when teaching the child how to self-administer insulin injections due to which influencing factor? a. Self-concept b. Self-esteem c. Developmental level d. Hierarchy of needs

Answer: c Even when the child understands about having the disease, she is less likely to understand the need for insulin therapy due to her emotional and comprehension level of development. A child's self-concept is not well established at this point, and self-esteem is not a major factor. Hierarchy of needs is not yet fully developed because the child depends on her parents for the basic level of needs. LO: 16.2

Which goal statement is appropriate for a patient with the nursing diagnosis of Acute Confusion? a. Patient will remember nurse's name. b. Nurse will remind patient of his or her name each shift. c. Patient will state name and date with each nursing encounter. d. Nurse will remind patient of name and date with each nursing encounter

Answer: c Goals are always patient centered and measurable and have a specified time frame. A patient goal would not include a nursing behavior. A confused patient would not be expected to remember different nurses' names but would be assessed for person, place, and time orientation with each encounter. LO: 31.5

2. The postanesthesia care unit (PACU) nurse is concerned about postoperative hemorrhage. Which clinical manifestation alerts the nurse to this problem? a. Incisional pain b. Elevated blood pressure c. Increased heart rate d. Bradypnea

Answer: c Increased heart rate is a clinical manifestation that occurs with hemorrhage. Incisional pain can be caused by the surgical procedure. When a person is hemorrhaging, the blood pressure decreases. With hemorrhage, tachycardia and tachypnea (not bradypnea) occur as the body tries to provide adequate tissue perfusion. LO: 37.8

What is the first action the nurse should take if an alert and oriented patient asks the nurse for personal contact information? a. Ask the patient why the personal information is needed. b. Report the interaction to the nursing supervisor immediately. c. State that it would not be appropriate to share that information. d. Change the subject and hope that the patient does not ask again.

Answer: c It is important for the nurse to immediately communicate that sharing personal contact information with patients is inappropriate and violates professional role boundaries. Asking "why" questions and changing the subject are nontherapeutic. Neither action will discourage the patient from further infringing on the nurse's personal right to privacy. Reporting the interaction to a supervisor may be helpful for preventing other nurses from experiencing similar requests; however, the first action taken by the nurse should be to maintain professional role boundaries. LO: 3.3

Which action by the nurse would be most important in developing a patient-centered plan of care for an alert, oriented adult? a. Providing a written copy of care options to the patient and family b. Collaborating with the patient's social worker to determine resources c. Listening to the patient's concerns and beliefs about proposed treatment d. Engaging the patient's family, friends, or care providers in conversation

Answer: c It is most important to involve the patient in developing realistic, attainable, patient-centered plans of care. Involving others in care planning is secondary to involving the patient unless the patient is cognitively impaired. LO: 8.7

Which statement is true regarding advance care planning and advance directives? a. Advance care planning applies only when the person is dying. b. Advance care planning should be done by family members of people who are incompetent. c. Discussion of advance care planning is a nursing responsibility. d. Advance directives should be kept in a safety deposit box until the person dies.

Answer: c It is the responsibility of the nurse to discuss advance care planning and advance directives with patients and their families; their benefits and limitations, how to complete an advance directive, and how advance directives can assist in decision-making at the end of life. Advance directives go into effect when a person has a terminal illness and is unable or incapable of making decisions for oneself. Advance directives are completed by people who are competent and have decision-making capacity. Advance directives should be discussed by the nurse with family members and the written documents should be given to family, health care providers, and those at institutions where health care is provided. LO: 42.7

. What nursing intervention is best when a patient is struggling with the decision to abort an abnormally developing fetus discovered during genetic testing in the first trimester of pregnancy? a. Recommend additional testing b. Refer the patient to an abortion clinic c. Listen to the patient's concerns d. Discuss regional adoption agencies

Answer: c Listening is the best option for the nurse when patients are considering ethical care decisions. Patients often need someone to listen to their verbalized concerns to sort out feelings about the situation and make decisions that are best for them. The patient's primary care provider is responsible for recommending further testing or making requested patient referrals. LO: 11.4

Which resource is most helpful when prioritizing identified nursing diagnoses? a. Nursing Interventions Classification (NIC) b. Gordon's functional health patterns c. Maslow's hierarchy of needs d. Nursing Outcomes Classification (NOC)

Answer: c Maslow's hierarchy of needs and the airway, breathing, circulation (ABCs) of life support are the most helpful tools in identifying priorities of care. Using functional health patterns is one method of organizing assessment data. NOC and NIC are resources for identifying outcomes and interventions to include in a patient's care plan after priorities have been established. LO: 8.2

The health care provider prescribes a transdermal medication. The nurse understands what feature of the transdermal route? a. It is inhaled into the respiratory tract. b. It is dissolved inside the cheek. c. It is absorbed through the skin. d. It is inserted into the vaginal cavity.

Answer: c Medicated patches or disks can be applied directly to the skin. The transdermal route of administration allows release and absorption of the medication through the skin over time, producing a steady drug level. Medications that are inhaled are aerosolized and not absorbed through the skin. The buccal route (inside the cheek) is a form of topical administration, but it is through the mucous membranes of the mouth, not transdermal (through the skin). Medications administered vaginally are topical, but they are applied to the mucous membrane of the vaginal wall, not to the skin. LO: 35.4

Which group is referred to as the sandwich generation? a. Older adults who are caretakers for their elderly parents b. Younger adults who are reexamining their life choices c. Middle adults who are caretakers for multiple generations of their family d. Younger adults who are changing employment constantly

Answer: c Middle adults who are caretakers for multiple generations of their family tend to be sandwiched in between the needs of children, grandchildren, and elderly parents, all of whom need their attention and care. This situation can lead to many health and financial problems for the middle adult. Although older adults caring for their elderly parent also may experience significant stresses, they would not be called the sandwich generation unless they also were caring for younger members of their family. Younger adults may change jobs or middle adults may reexamine choices they have made, but these factors do not reflect the concept of the sandwich generation.

The nurse is caring for a patient who states they have not been able to sleep while in the hospital. Which action would be a priority to implement? a. Administer a sleeping medication with the evening meal. b. Restrict visitors for the patient in the evening. c. Decrease noise around the patient during the night. d. Offer a hot drink of regular tea at bedtime.

Answer: c Noise is a primary cause for disturbed sleep in the hospital. Administering sleeping medications with the evening meal is too early to help the patient sleep throughout the night. Restricting visitors may be helpful if the patient requests it, but visitors often provide emotional support and reassurance to the patient, which helps with relaxation. Regular tea contains caffeine, which is not helpful in sleep promotion. LO: 33.6

The nurse is caring for a religious patient who is going to surgery the next day. The patient states that she is afraid and asks the nurse to pray with her, although the nurse is not religious. What is the most appropriate response by the nurse? a. "I am not confident praying, but I will think about you tomorrow." b. "I need to take care of other patients right now, but I will be back." c. "I am uncomfortable praying. May I call the chaplain for you?" d. "I don't do that. Nurses are not allowed to do that at our hospital."

Answer: c Offering to call the chaplain because the nurse is uncomfortable praying is the best option. Avoiding the subject or focusing on the nurse's feelings or needs is not appropriate and will not provide for the patient's stated spiritual need. LO: 22.6

A hospitalized patient experiences a sharp, stabbing pain while visiting with his spouse. Both the patient and his wife become very concerned, and the patient's call light is activated. What referent initiated the communication between the patient and the nurse? a. Interaction between the patient and his wife b. Concern on the part of the patient's spouse c. Pain experienced by the patient d. Activation of the call light

Answer: c Pain is the referent that initiated the communication process. The interaction between the patient and his wife was the result of the patient's pain, as was the concern of the patient's spouse. The call light could be considered a channel through which the patient's interaction with the nurse began. LO: 3.1

When developing treatment plans, which assumption should the nurse consistently make about individual clients within vulnerable populations? a. Educational levels are minimal. b. Economic resources are strong. c. Personal beliefs are important. d. Support systems are extensive.

Answer: c Personal beliefs of clients within a vulnerable population are always important. They are a key to how people will respond to care that is offered. Educational levels, economic resources, and support systems vary dramatically by individual clients within vulnerable populations. LO: 23.4

Written instructions showing pictures of the steps necessary to test blood glucose, along with demonstration and a return demonstration of the steps, would most benefit which learners? Affective VARK Psychomotor Cognitive

Answer: c Psychomotor learning involves physical movement and the use of motor skills such as demonstration and return demonstration. The affective domain involves emotion, and the cognitive domain is memorization and recall. VARK (verbal, aural, read/write, kinesthetic) refers to a method of assessing learning style. LO: 14.4

Which statement by a patient best illustrates reflection on a spiritual need? a. "My husband told me what to do about this situation, and I'm sure he's right." b. "There is little I can do now to change my circumstances. I just need to adapt." c. "I need to think a little more about how I feel about undergoing this treatment." d. "Whatever the physician wants to do is fine. I don't have much of an option."

Answer: c Reflection requires intentional thought about a situation to determine how it affects or is affected by the person's beliefs and values. Simply having someone tell the person what to do is not reflection. Adopting a fatalistic attitude or accepting the decision of a physician without inquiry is not reflection either. LO: 22.7

Which intervention would be most appropriate for the nurse to include in the care plan for a patient who is experiencing constipation and increased heart and respiratory rates? a. Time management b. Decreased grain intake c. Relaxation therapy d. Regimented exercise

Answer: c Relaxation therapy typically lowers the person's heart and respiratory rates while increasing gastric motility. Not enough information is provided to indicate the need for time management. Both decreased grain intake and regimented exercise may exacerbate the patient's problems. LO: 32.7

How do people who participate in organized religion differ from nonreligious people? a. Religious people are healthier than spiritual people. b. Religious people are more spiritual than nonreligious people. c. Religious people express their spirituality through faith traditions. d. Religious people have spiritual practices, whereas nonreligious people do not have spiritual practices.

Answer: c Religious people express their beliefs through faith traditions. Research has not indicated that religious people are healthier or more spiritual than those who do not participate in organized religion. Nonreligious people may practice various spiritual disciplines. LO: 22.2

A patient diagnosed with an aggressive cancer is estimated to have 6 months to live. Two months later, the patient's wife calls the nurse's office because she is upset that her husband has taken up motorcycle racing and has already been injured twice. The nurse knows that the patient is experiencing a behavioral change in which factor due to the prognosis of his illness? a. Spirituality b. Physical attributes c. Self-concept d. Personal affect

Answer: c Self-concept is profoundly affected by the diagnosis of a terminal disease. People often try to reinvent themselves and behave in an uncharacteristic manner in the face of such a diagnosis. LO: 16.6

The nurse is teaching a patient about how to take a sublingual nitroglycerin tablet. Which statement by the patient best demonstrates understanding of the teaching? a. "I will take the tablet with plenty of water." b. "I will place the tablet inside my cheek." c. "I will put the tablet under my tongue." d. "I will take the tablet while I am eating."

Answer: c Sublingual medications are placed under the tongue, where they are absorbed quickly into systemic circulation. Medications placed in the cheek are delivered by the buccal route. Sublingual medications should be taken without water or food. LO: 35.4

The nurse is assigned to administer medications to a patient on a unit that has bar-code medication administration (BCMA). Which action is proper for the nurse to follow? Open the medication packages at the nurses' station. Ask the patient to verify his or her address. Scan the nurse's ID, the patient's ID, and the code on the medication package. Ask the patient to name two patient identifiers.

Answer: c The BCMA system scans the nurse's ID, the patient's ID, and the medication package to ensure that the proper drug is given to the correct patient. Asking the patient's address or two random identifiers that the patient may not be aware of would be inappropriate. Proper protocol for administration is to open the medication packages at the bedside. Use of a scanning device requires the medication to still be in the package while scanning to ensure that it is the proper medication. LO: 15.3

Which description is true about the Nursing Minimum Data Set (NMDS)? An admission assessment tool A discharge summary The core nursing data for collection across all sites An organization of nursing diagnoses

Answer: c The NMDS is a standardized collection of essential nursing data used by nurses to promote consistent, understandable documentation. Although standard terminology may be used during admission and discharge, these tools are not a description of NMDS. Nursing diagnoses are organized using one of several taxonomies, including ICNP, and NANDA-I. LO: 15.5

The nurse is assessing the intravenous (IV) site in the right antecubital and notices that the area about 1 inch around it is cool, swollen, firm, and tender to touch. Which action should the nurse take first? Take patient's temperature. Apply an ice pack to site. Stop infusion and remove IV catheter. Call the primary care provider immediately.

Answer: c The area around an IV infiltration is cool, swollen, firm, and tender to touch. The first intervention to take for an infiltrated IV is to stop the infusion and discontinue the IV/remove the catheter. Applying cold compresses may be appropriate for hyperosmolar fluids but only after the IV infusion has been stopped. Taking the temperature would be an assessment to make if the complication of infection is suspected. The primary care provider does not need to be notified unless grade 3 or 4 infiltrations are noted (> 6 inches edema). LO: 39.6

The nurse is caring for a patient who is unable to hold a cup or spoon. How should the nurse administer oral medications to the patient? a. Crush the pills and mix them in pudding before administering. b. Ask the pharmacist to change all of the medications to a liquid form. c. Use a small paper cup to place the pills into the patient's mouth. d. Place the pills on the table and have the patient take the pills by hand.

Answer: c The nurse assists the patient by using a small cup to place the medications into the patient's mouth. Using a cup rather than the nurse touching the pills maintains medical asepsis. Crushing the pills with food is appropriate for a patient with dysphagia, not for a person who is unable to hold objects. Liquid medications may be used for some swallowing difficulties or to replace very large pills. It is not safe to leave medications on a table, because they can be contaminated, lost among other items, dissolve in spilled liquids, or be missed. Leaving the pills on a table does not address this patient's difficulty in holding objects and is not aseptic. LO: 35.9

. The nurse reviews a primary care provider's order and finds that the medication amount is greater than the standard dose. What action should the nurse take? a. Give the standard dose rather than the one that is ordered. b. Consult with the nursing supervisor to get a second opinion. c. Call the primary care provider to discuss the order in question. d. Administer the medication as ordered by the primary care provider.

Answer: c The nurse is responsible for understanding and further investigating a medication order that falls outside an acceptable standard. Ultimately, nurses are responsible for their own actions despite a PCP's written order. It is not within the nurse's scope of practice to alter a medication order. However, the nurse has the right to refuse to give a medication and the responsibility to question orders, as necessary, to ensure patient safety. Consulting with the nursing supervisor for a second opinion is unnecessary. LO: 35.5

While performing a physical assessment on a female patient, the nurse finds several bruises on the patient's inner thighs that are in various stages of healing and suspects that the patient may be a victim of sexual abuse. What should be the nurse's first action? a. Refer the patient to a sexual counselor. b. Tell the patient about the safe house for women. c. Ask the patient to describe how she got the bruises. d. Report the abuse immediately to the proper authorities.

Answer: c The nurse's first action is to gather more data that can confirm or negate the suspicion of sexual abuse. The other actions also could be appropriate after additional information is obtained. LO: 24.10

What is the best activity for a hospitalized school-age child to encourage continued appropriate development? a. Watching favorite television shows for 2 hours per day b. Keeping a journal of feelings while in the hospital c. Working on a paint-by-number project that can be completed in an afternoon d. Playing a favorite video game each afternoon

Answer: c The school-age child is in Erikson's stage of industry. He needs to work on projects that build a sense of accomplishment. A painting project that can be completed in one afternoon gives a sense of accomplishment. Although the other options are activities, they do not contribute to the primary developmental task of the school-age child—developing a sense of industry. LO: 17.1

How is the toddler's need for autonomy best met? a. The parents' consistently meeting the child's needs b. Encouraging imaginative play c. Allowing the child limited choices d. Promoting experimentation to determine cause and effect

Answer: c The toddler's need for autonomy can best be supported by allowing the child choices within limits. Parents consistently meet the child's needs during the infant period. In later developmental stages, imaginative play is encouraged, and experimentation is promoted to determine cause and effect. LO: 17.5

Which statement best contributes to the nurse's documentation of assessment of patient status in the patient's medical chart? "Patient had a good day with minimal complaints. Patient was pleasant and cooperative during morning care." "Patient complained that the nurse didn't come quickly enough when she pressed the call button." "Patient rated pain 7/10 at 7:45 a.m. Received pain medication at 8 a.m., reporting pain 3/10 at 8:30 a.m." "Patient was grumpy today, even after administration of pain medication, a back massage, and a nap."

Answer: c This entry is concise, complete, and objective. It gives exact times, pain levels, and nursing interventions performed. Using terms such as good or grumpy are subjective judgments or opinions and should be avoided. Stating a patient complaint would be okay if it listed specific times of occurrence, nursing assessment performed, and the nursing interventions performed to correct the issue. LO: 10.3

Which statement is correct concerning the implementation of computerized provider order entry (CPOE)? The unit secretary transcribes the Primary Care Provider's (PCP's) orders into the computer. The nurse must ensure that orders go to the appropriate departments. Primary Care Provider (PCP) orders go directly to the appropriate department. Handwriting legibility is a major problem.

Answer: c Use of CPOE enables orders to go directly to the appropriate department, decreasing the potential for errors. There is no transcription of orders and no need for someone to send the orders to the correct department. Because the orders are typed into the computer, handwriting legibility is not an issue. LO: 15.3

When using a stress assessment tool with a patient from another culture, what factors must the nurse take into consideration? (Select all that apply.) a. Specific methods of managing stress are revealed in using stress assessment tools. b. Stress assessment tools should be used only for persons living in North America. c. Stress assessment tools may not be appropriate for all people of all ages. d. Resistance resources become evident when stress assessment tools are analyzed. e. Adaptations may need to be made to the assessment tool based on circumstances.

Answer: c, e It is not possible to use stress assessment tools in some situations. Stress assessment tools must be adapted to specific age groups, cultures, and circumstances to be most effective in gathering pertinent data. Stress assessment tools identify only stressors that the person is experiencing and not methods of managing stress or the person's resistance resources. LO: 32.4

Processes used in management parallel the nursing process. Which sentence describes a nurse using a management principle paralleled with the nursing process? Planning is demonstrated when the nurse motivates others. Directing is demonstrated when the nurse plans care for the patient. Organizing is demonstrated when the nurse coordinates care for patients. Controlling is demonstrated when the nurse tells other staff members what to do.

Answer: cAccording to Fayol, the functions of managers are planning, organizing, directing, and controlling. The nurse demonstrates organizing by coordinating the care delivered to patients. Planning involves goal setting, assessment, setting the plan and acting on it. Direction involves the ability to motivate others toward a common goal and includes good communication skills. Controlling involves comparing expected results of the planned work with actual results. LO: 12.3

The nurse is providing home care to a 62-year-old woman who was recently diagnosed with insulin-dependent diabetes mellitus. What is the most important reason for the nurse to document the teaching session? The patient's insurance company requires documentation. The nurse's employer requires documentation of home care sessions. Other members of the health care team need to know the patient's progress. Insulin is a potentially dangerous medication and needs to be documented.

Answer: cAlthough the remaining options may be true, the primary reason for specific documentation of a patient's progress in a teaching plan is to ensure that other nurses or members of other disciplines can pick up the teaching plan and know precisely what the patient has accomplished and where to begin additional sessions. LO: 14.10

Which action by the day-shift nurse provides objective data that enables the night-shift nurse to complete an evaluation of a patient's short-term goals? a. Encouraging the patient to share observations from the day b. Leaving a message with the charge nurse before shift change c. Documenting patient assessment findings in the patient's chart d. Checking with the pharmacist regarding possible drug interactions

Answer: cDocumentation of assessment findings is the only objective form of data listed as an option that can support the night nurse in evaluating whether the patient achieved short-term goals. Patient observations are subjective. Leaving a message with the charge nurse produces secondary subjective data and checking for drug interactions is unrelated to the evaluation process. LO: 9.8

The nurse feels that the results of a recent literature search and analysis about handwashing should be implemented in the entire hospital system. With whom would the nurse be required to collaborate? a. Colleagues caring for patients in her unit b. Colleagues in the community c. Administrators at the hospital d. Others in her department

Answer: cImplementation would require collaboration with hospital administrators to influence the larger practice environment of the entire hospital. Collaboration with colleagues in the community would have no effect on the hospital system. Collaboration with the department or colleagues in the unit would not allow for hospital-wide implementation. LO: 13.6

Which statement best describes for new parents how and when children develop first-order beliefs? a. During infancy, and once developed, such beliefs seldom change b. From life experiences during the toddler and preschool years c. Throughout life from firsthand experiences and information provided by authority figures d. From teen and young-adult peer interaction and mentorship of professional role models

Answer: cIndividuals develop first-order beliefs beginning in childhood and continue to acquire them throughout life from firsthand experiences and what they are told by various authority figures. First-order beliefs are acquired throughout life and not just in infancy, the first years of life, or adolescence. They form as the result of life experiences and from information provided by people perceived as having authority. LO: 2.2

A nurse states that she believes in the dignity of each patient. At break, she is overheard talking about a patient in a persistent vegetative state as a "lump." This represents an inconsistency in which quality of an effective leader? Dedication Magnanimity Integrity Humility

Answer: cIntegrity refers to the alignment of stated values and actions. Dedication is the ability to spend the time to accomplish the task. Magnanimity means giving credit where credit is due. Humility is the ability to recognize that no one person is superior to another. LO: 12.2

Which statement most closely reflects the differences between nurse leaders and managers? Nurse leaders are always in formal positions of authority. Nurse managers use transactional principles to accomplish goals. Nurse leaders rely primarily on interpersonal skills to accomplish goals. Nurse managers rely on supervisors for accountability and responsibility.

Answer: cLeaders influence others to effect change. They rely on personal characteristics to convince others that what they envision is worthwhile. Managers get their power from formal positions. Leaders may or may not be in formal positions of authority. Transactional leadership employs reward and punishment to gain the cooperation. Nurse leaders use a variety of leadership and management styles. Nurse managers maintain accountability and responsibility for their decisions. LO: 12.1

Which nursing theory of care describes how the nurse's presence in the nurse-patient relationship goes beyond the physical and material world, facilitating the development of a higher sense of self by the patient? a. Swanson's Theory of Caring Processes b. Madeline Leininger's Cultural Care Theory c. Watson's Theory of Human Caring d. Boykin and Schoenhofer's Theory of Nursing as Caring

Answer: cOne of the major concepts of Watson's Theory of Human Caring is described in the stem of the question. Watson's theory is based on a holistic paradigm in which both the nurse and the patient transcend time and the physical and material world. Swanson's theory focuses on practical ways that the nurse can help the patient using the five caring processes. Leininger's theory focuses on maintaining and preserving the patient's cultural practices and ways of living but never mentions transcending beyond the physical world. Boykin and Schoenhofer's theory stresses living in caring and growing in caring.

Which phrase best represents a related factor in a problem-focused nursing diagnosis? a. Unsteady gait requiring the assistance of two people b. Redness and swelling around the incision site c. Ineffective adaptation to recent loss d. Patient complaint of restlessness

Answer: cRelated factors are broad statements that indicate the cause for the defining characteristics, which are signs or symptoms identified from collecting the patient's data. Redness and swelling, unsteady gait, and complaint of restlessness are specific defining characteristics that would be clustered with other data to support the existence of a problem-focused or health promotion nursing diagnosis. LO: 7.3

After admitting a homeless patient to the floor, the nurse tells a colleague that "homeless people are too dumb to understand instructions." What action should the colleague take first? a. Ignore the nurse's prejudicial comment without responding. b. Offer to trade assignments and care for the homeless patient. c. Ask the nurse about the patient's personal history assessment data. d. Challenge the nurse's thinking, pointing out the ability of all people.

Answer: cThe colleague should first ask the nurse to share information about the patient's background. This should encourage the nurse to consider the feelings and values of the patient and, hopefully, help the nurse view the patient as a total individual. Ignoring the statement, offering to change assignments, or challenging the nurse's statement does not promote an enhanced nurse-patient relationship and may prevent the nurse from professional growth or make the nurse defensive. LO: 2.2

Health care workers are discussing a diverse group of patients respectfully and are being responsive to the health beliefs and practices of these patients. What important aspect of nursing professional practice are they exhibiting? a. Autonomy b. Accountability c. Cultural competence d. Autocratic leadership

Answer: cThe nurse and other health care workers are exhibiting cultural competence by being responsive to patients' health beliefs and practices that are influenced by each patient's culture. Autonomy is being independent and self-motivated. Accountability is accepting responsibility for one's own actions and omissions. An autocratic leader exercises strong control over subordinates.

A nurse makes a medication error, immediately assesses the patient, and reports the error to the nurse manager and the primary care provider (PCP). Which characteristic of a professional is the nurse demonstrating? a. Autonomy b. Collaboration c. Accountability d. Altruism

Answer: cThe nurse is demonstrating accountability by taking responsibility for the error and reporting it after an initial assessment of the patient. Criteria of a profession include altruism (public service over personal gain), autonomy (independence), accountability, and diversity; however, in this case, the nurse is demonstrating accountability. Although collaboration is important for the health care team, it is not a criterion for a profession.

Which task may the registered nurse safely delegate to unlicensed assistive personnel without prior intervention? a. Ambulating a patient with ataxia and new right-sided paresthesia b. Feeding a patient with cerebral palsy who recently aspirated c. Transporting a patient to the hospital entrance for discharge d. Administering prescribed programmed medications

Answer: cTransporting the stable patient for discharge can be delegated immediately to unlicensed assistive personnel (UAP). A patient with new neurologic symptoms needs to be assessed before being ambulated. Patients who have recently choked need to be evaluated for their ability to swallow before being fed. Administering medication is not within the UAP's scope of practice and can never be delegated to UAP. LO: 9.2

Which intervention would be most important for the nurse to include in a patient's care plan if the patient is unable to complete activities of daily living without becoming fatigued? a. Instruct the patient to shower and shave simultaneously. b. Discourage the patient from bathing while hospitalized. c. Encourage the patient to rest between bathing activities. d. Ask the patient's spouse to assist with all bathing.

Answer: cWhen patients are unable to complete their personal care without fatigue, it is best to encourage them to rest between activities. All patients should be encouraged to wash during hospitalization and to complete as much of their personal care as independently as possible. Patients who tire easily should not be encouraged to shower and shave simultaneously but should space out personal care while seated. LO: 9.1

Which core competency of advanced practice registered nurse (APRN) is a nurse educator exhibiting when counseling a student nurse on the unit in therapeutic communication techniques? a. Leadership b. Ethical decision making c. Direct clinical practice d. Expert coaching

Answer: d A nurse educator who is teaching and counseling students is practicing expert coaching and guidance. A nurse educator with a master's degree practices the other competencies of leadership and ethical decision making in other situations. Although a nurse educator may also work as a nurse involved in direct patient care, this is not part of the educator role.

Which statement is the best resource for the nurse to use when determining appropriate nursing care for a transsexual patient? a. Gender identity is altered by acute psychosis. b. Sexual attraction is to individuals of both genders. c. Gonadal gender, internal organs, and external genitals are contradictory. d. Anatomy associated with sexual identity is not consistent with gender identity.

Answer: d A transsexual person's sex organs do not match gender identity. Being a transsexual is not a psychosis. Transsexuals usually are attracted to persons of the gender opposite their own gender identity. Gonadal gender and internal and external organs are not in contradiction. LO: 24.3

A 65-year-old male patient has been a one-pack-per-day smoker for 40 years. He was recently diagnosed with chronic obstructive pulmonary disease (COPD) and would like to attend a smoking cessation class. The nurse recognizes smoking cessation as which level of prevention for this patient? a. Primary prevention b. Secondary prevention c. Statutory prevention d. Tertiary prevention

Answer: d According to the stages for disease prevention, primary prevention is implemented for the absence of disease, secondary prevention applies to the early stages of disease or recently diagnosed risk factors, and tertiary preventive care is offered for permanent and irreversible disease. In this case, even though the patient's COPD was recently diagnosed, the care would be considered tertiary prevention because his condition is permanent. LO: 16.4

What action should nurses who demonstrate accountability take if they forget to administer a patient's medication at the ordered time? a. Document the medication as refused by the patient. b. Administer the medication as soon as the error is discovered. c. Record the medication as given after making sure the patient is okay. d. Follow the administration and documentation procedures for medication errors.

Answer: d Agency procedures must be followed after every medication error. Care must be taken to adhere to medication administration recommendations and documentation requirements to legally record the incident and provide patient safety. Documenting that the patient refused or already took the medication when that is not factual is illegal and unethical regardless of the patient's condition. Administering the medication as soon as the error is discovered may not be recommended depending on the medication's potency and frequency of administration. LO: 11.2

If a student nurse overhears a peer speaking disrespectfully about a patient, nurse, faculty member, or classmate, what is the most ethical first action for the student nurse to take? a. Discuss the peer's actions during group clinical conference b. Ignore the initial occurrence and observe if it happens again c. Report the actions of the classmate to the clinical instructor d. Speak to the peer privately to prevent further occurrences

Answer: d Alerting the peer who has acted disrespectfully in a private setting is the most professional way to approach this situation. It is never appropriate for a professional to belittle or reprimand a peer in front of others which may be perceived as a form of bullying. Ignoring disrespectful behavior may only perpetuate its occurrence. Seeking help from a clinical instructor would be appropriate if the peer does not respond to the initial intervention from the fellow student. LO: 11.3

. Which method is the most accurate way to determine the pain level of a patient who is alert and oriented? a. Evaluate whether the patient is crying or grimacing. b. Assess the patient's heart rate and blood pressure. c. Consider the seriousness of the patient's condition. d. Ask the patient to describe the pain and rate its level.

Answer: d Because pain is defined as what a patient says it is, a patient's report based on the pain scale is currently the most accurate way to determine the pain level of a cognitively alert patient. Crying or grimacing may be considered on a noncognitive scale for a nonverbal patient. Vital signs and the patient's condition contribute to a pain assessment, but they may not be the most accurate determinants. LO: 36.5

A 75-year-old male patient reports decreased frequency of sexual intercourse, although he does not express dissatisfaction or difficulty. He seems a little embarrassed by the discussion but is engaged and asks some questions. Which nursing diagnosis (problem) does the nurse determine is most appropriate for this patient? a. Impaired Sexual Functioning b. Disturbed Body Image c. Difficulty Coping d. Lack of Knowledge

Answer: d Because the patient can discuss the topic of reduced sexual frequency without expressing difficulty or dysfunction, manages any embarrassment, is engaged in the conversation, and is able to ask questions, the most appropriate nursing diagnosis is Lack of Knowledge. The nurse would develop a care plan to include age-appropriate discussion topics and/or patient education materials focused on the patient's physical condition, social situation, and his questions. The patient's collective behaviors do not describe Impaired Sexual Functioning, Disturbed Body Image, or Difficulty Coping. LO: 24.11

The nurse is caring for a patient with decreased sensation in the lower extremities. Which precaution does the nurse advise the patient to take? a. Use heat to warm hands during cold weather. b. Go barefoot at home to prevent blisters from shoes. c. Soak feet in cold water daily to decrease swelling. d. Test the bath water temperature to prevent burning injuries.

Answer: d Because the patient may not be able to feel the temperature of the water, using a thermometer will prevent burns. The use of heat and cold is contraindicated in patients with tactile deficits because they would not be able to feel whether the therapy was too hot or cold. The patient should wear good-fitting shoes around the house to prevent foot injury. LO: 31.6

The nurse has been caring for a patient who has just died. What is the preferred outcome in caring for the body after death? a. Make sure that the body is sent to the morgue within an hour after death. b. Have the family members participate in the bathing and dressing of the deceased. c. Notify in person or by phone all family and team members immediately after the patient's death. d. Demonstrate respect for the body and provide a clean, peaceful impression of the deceased for the family.

Answer: d Demonstrating respect for the deceased maintains the dignity of that person and also can help the family in the grieving process. Proper positioning of the body and covering the body appropriately will promote a peaceful impression of the deceased for the family. Family often will request time with the deceased, and it is not necessary to place a time frame of 1 hour for the arrival of the body at the morgue. Family members may want to participate in bathing and dressing their loved one after death, but this should be their choice. It is not the responsibility of the nurse to notify all family members and team members of the death immediately because the nurse will need to provide care to the family and determine what type of assistance they need in notifying family members. LO: 42.7

The nurse knows that which patient is an example of the Wear-and-Tear Theory of Aging? a. A patient who is dying of cancer at age 35 b. A 55-year-old who runs half-marathons c. A patient with depression and suicidal thoughts who is 65 d. An 88-year-old with heart failure, kidney failure, and osteoarthritis

Answer: d Dr. Weisman believed that aging occurred because of overuse and abuse to body and cells, and that this takes place on the cellular and organ-systems level. An 88-year-old with multiple body systems failure and damage would be exhibiting signs of wear and tear. A 35-year-old patient dying of cancer would not have overuse. A 55-year-old running in half marathons is not exhibiting wear and tear, although signs of overuse may emerge later in life. A 65-year-old person with depression is not exhibiting wear and tear. Emotional stressors are only one group of factors that can lead to aging and are not a component of the Wear-and-Tear Theory of Aging. LO: 18.1

How does malnutrition compromise wound healing? a. There is increased stress on the wound. b. Blood supply to the wound is increased. c. It causes patients to be energized and overexert. d. It can increase the risk of infection.

Answer: d Malnutrition can increase the risk of infection, which can interfere with the formation of granulation tissue and new tissue growth, delaying wound healing. Malnutrition does not directly cause stress on the wound. Blood supply to the wound may be poor, not increased, which can lead to delayed wound healing. Malnourished patients do not feel energized. Poor nutrition contributes to malnourished cells, which leads to reduced energy. LO: 37.2

While conducting a preoperative health assessment, the nurse is informed about a patient's preexisting heart problem. What postoperative interventions should be included in the plan of care for this patient to address the heart issue? a. Perform a systematic head-to-toe assessment every 4 hours. b. Monitor breath sounds and oxygen saturation. c. Administer pain medications as needed. d. Monitor the electrocardiogram (ECG), apical pulse, and capillary refill.

Answer: d Monitoring the patient's ECG, pulse, and capillary refill allows the nurse to assess the cardiovascular system. A head-to-toe assessment every 4 hours does not provide specific information for a focused assessment of the heart. Monitoring breath sounds and oxygenation saturation provides information about the respiratory system. Administering pain medication is necessary but does not give the nurse specific information about the heart. LO: 37.7

Which statement is most accurate regarding symbolic expression? a. Skills confidence can be shared most effectively by nurses through wearing distinctive clothing. b. Clothing choices by a hospitalized patient rarely reflects the patient's economic resources. c. Make-up use by a patient is unnecessary for any reason during hospitalization. d. Nondramatic make-up use and minimal accessorizing by nurses demonstrates professionalism.

Answer: d Nurses demonstrate professionalism by adhering to institutional dress codes that require minimal accessorizing and cosmetic use. Wearing distinctive clothing is not linked to skills confidence. Clothing choices often reflect the economic resources of an individual, and make-up use by a hospitalized patient is a personal preference that should be honored. LO: 3.2

Which of the listed basic needs identified by Maslow must be addressed first when providing nursing care? a. Self-esteem b. Love and belonging c. Self-actualization d. Nutrition and elimination

Answer: d Nutrition and elimination must be addressed first before concerns about self-esteem, love and belonging, and self-actualization, according to Maslow's hierarchy of needs. According to Maslow, the lower-level needs must be fulfilled and maintained before the higher-level needs can be met. LO: 16.2

A preschooler's mother is concerned because her child behaves in a mean fashion toward her younger brother. The mother states, "She acts like she has no sympathy for him!" What is the nurse's best response? a. "She is very young to exhibit sibling rivalry." b. "What does her brother do to her to make her act this way?" c. "Do you fight at home? She is probably imitating you." d. "Preschoolers are not capable of putting themselves in another's place."

Answer: d One characteristic of preschool thought is that it is egocentric. That is, preschoolers are not yet able to see a situation from another's point of view. Sibling rivalry commonly appears in the preschool period. The brother's behavior has no bearing on the mother's expressed concern. The child is not necessarily imitating adult behavior but is displaying appropriate developmental characteristics. LO: 17.6

Which notation is most appropriate for the nurse to include in a patient's chart regarding evaluation of the goal "Patient will ambulate three times daily in the hallway before discharge without shortness of breath (SOB)"? a. Goal not met; patient states he is tired. b. Goal not met; patient ambulated three times in room. c. Goal met; patient ambulated three times in the hallway. d. Goal met; patient ambulated three times in the hallway without SOB.

Answer: d Option d is the only notation that indicates whether the goal was met and how all of the outcome criteria were attained. LO: 9.8

The nurse is providing care to an 88-year-old male patient who just returned from the recovery room after a right hip replacement. The nurse plans to teach the patient prevention techniques for deep vein thrombosis. What is the best time to provide teaching? Do it right before the patient's next intravenous pain medication. Wait until tomorrow morning because he is in too much pain today. Leave written materials on his over-the-bed tray that he can read at his convenience. Wait until 10 to 15 minutes after his next intravenous pain medication.

Answer: d Patients in pain are unable to focus on learning. Waiting 10 to 15 minutes after the administration of intravenous pain medication allows it to provide relief, but the patient is not sedated or resting soundly. Waiting until the following day is inappropriate because early intervention and prevention are necessary to avoid the development of deep vein thrombosis. Leaving important information where it can be easily covered up, set aside, or overlooked is not an effective method of patient education. The nurse should remember the concepts of health literacy and consider the potential effects of visual impairments, reading ability, and pain level in ensuring patient comprehension. LO: 14.10

Which recommendation in the home-going instructions is appropriate for a patient with damage to the chemoreceptors of the upper nasal passages? a. Arranging for lighted signals on doorbells and telephones b. Obtaining a thermometer for testing bath water temperature c. Installing amplification devices on televisions, doorbells, and telephones d. Scheduling yearly safety checks of gas hot water heaters and furnaces

Answer: d Patients with damage to the chemoreceptors of the nasal passages may not be able to smell noxious fumes from household appliances. Lighted signals and amplification are interventions for a person with auditory deficits. Testing the bath water temperature is important for patients with tactile deficits. LO: 31.6

Which action would the nurse undertake first when beginning to formulate a patient's plan of care? a. List possible treatment options b. Identify realistic outcome indicators c. Consult with health care team members d. Rank patient concerns from assessment data

Answer: d Prioritizing or ranking patient needs precedes the identification of outcome indicators, consulting with team members, or consulting with interdisciplinary team members. LO: 8.1

A patient who had a hysterectomy 3 days ago says to the nurse, "I no longer feel like a real woman." Which response by the nurse would be most appropriate? a. "Don't worry about that. The feeling will probably go away." b. "You should talk to your doctor about how you feel." c. "I don't blame you. I would feel like half a woman also." d. "I hear your concern. Tell me more about your feelings."

Answer: d Providing an opportunity for communication with an open-ended response encourages the patient to discuss concerns. Telling the patient not to worry is dismissing those concerns and will hamper discussion. Agreeing with the patient also is nontherapeutic and does not foster dialogue. Telling the patient to talk with the doctor stops the chance of conversation and reduces the nurse's role in helping the patient express feelings. LO: 24.13

The nurse is caring for a 45-year-old woman who is a breast cancer survivor. What activity associated with her cancer experience will promote this patient's spiritual well-being? a. Attending church every week b. Making sure that she follows her medication regimen c. Arranging for the genetic testing on family members d. Speaking about her cancer experience to increase breast cancer awareness

Answer: d Speaking about her experience helps promote meaning and purpose in life. Church attendance does not guarantee time of spiritual reflection related to her breast cancer experience. Treatment and genetic testing assist in physical treatment and risk identification but do not necessarily relate to finding meaning and purpose. LO: 22.1

What is the best method for the nurse to ensure that a Croatian patient's nutritional needs are met during hospitalization? a. Preorder a diet that is consistent with the typical Croatian patient's dietary preferences. b. Ask a Croatian co-worker for ideas on what would be best to order for the patient's meals. c. Request that a variety of dietary entrees be provided to the patient to offer options. d. Check with the patient on admission to determine dietary limitations and preferences.

Answer: d The best way to provide for a patient's dietary needs is to ask the patient for personal preferences, limitations, allergies, and typical dietary intake. Preordering, checking with a co-worker, or ordering a variety of options without input from the patient first does not reflect patient-centered care. LO: 21.5

The outpatient clinic nurse develops a plan of care focusing on diet, exercise, and glucose monitoring for a preteen recently diagnosed with early-onset type 2 diabetes. On what type of interventions has the nurse based the client's care plan? a. Primary b. Proactive c. Secondary d. Tertiary

Answer: d The nurse has incorporated tertiary interventions, because the client has already been diagnosed with an illness. Tertiary interventions are implemented once a disease occurs to prevent further deterioration. LO: 23.2

The nurse is caring for a patient scheduled for a breast reduction to decrease pain in her back. How is this operation classified according to the degree of urgency? a. Urgent b. Emergency c. Emergent d. Elective

Answer: d The patient is choosing to have this procedure; therefore, it is elective surgery. Urgent surgery is performed when it is necessary for the patient's health. It is commonly performed within 24 hours of the diagnosis. Emergency surgery is performed immediately to preserve life, body parts, or function. Emergent surgery is the same as emergency surgery. It must be performed immediately to preserve life, body parts, or function. LO: 37.2

How can the nurse best address the religious needs of patients who are Jewish? a. By documenting the need for a kosher diet b. By allowing time for prayer before each meal c. By inquiring about Sabbath religious practices d. By asking about religious practices that might affect care

Answer: d There are a variety of levels of observance within the Jewish community. It is best to ask patients about what practices will affect their care. Do not assume that all persons of faith are equally observant of religious traditions. LO: 22.4

After reading various research articles and reviews on a subject, the nurse designs a practice change based on the literature. What stage of EBP is this? a. Maintaining the change b. Implementing the change c. Evaluating and critically appraising d. Synthesizing the evidence and developing a plan

Answer: d This stage requires the nurse to synthesize all the information found and develop a plan for a practice change. LO: 13.5

A 25-year-old female patient demands that her mother or father be present during all blood testing. Which defense mechanism could the nurse document as being used by this patient? a. Sublimation b. Repression c. Projection d. Regression

Answer: d This young adult patient is reverting back to behavior consistent with an earlier stage of development, which is the defense mechanism of regression. Sublimation is channeling unacceptable emotions into acceptable actions. Repression involves blocking unacceptable thoughts from consciousness, and projection attributes one person's desires or traits to another person. LO: 32.1

. At a routine clinic visit, an athlete training for a major sports event reports difficulty sleeping that is affecting the training schedule. What would be the best recommendation by the nurse for this patient to promote sleep? a. Increase the use of electrolyte-enriched drinks to increase stamina. b. Obtain a short-term prescription for sleeping medications. c. Plan to arise later in the morning to accommodate sleep changes. d. Avoid vigorous exercise for at least 2 hours before bedtime.

Answer: d Vigorous exercise in the hours before bedtime will cause stimulation that prevents sleep. Adjusting the training schedule to account for this effect is the preferred first step for improving the athlete's sleep rather than starting medications that may affect alertness during the day. A regular sleep schedule is preferred to maintain sleep promotion, including getting up at the same time each day no matter when bedtime occurred. LO: 33.6

What action should a nurse take if a patient who needs to sign an informed consent form for nonemergency surgery appears to be under the influence of drugs or alcohol? a. Contact the physician to see what should be done. b. Ask the patient's spouse to sign the informed consent form. c. Request permission to bypass the need for a signed consent form. d. Wait to have the informed consent form signed when the patient is alert and oriented.

Answer: d When a normally competent patient is assessed to be under the influence of alcohol or drugs, it is the nurse's responsibility to delay a nonemergency procedure until legal informed consent can be obtained from the patient. Only in the case of an emergency is it possible to obtain informed consent from a spouse or designated power of attorney for a temporarily impaired adult. It is not necessary to contact the physician for guidance on what should be done because there is an established legal procedure to follow. The nurse should contact the surgeon who is scheduled to perform the surgery and the operating room staff regarding the need to delay due to the patient's status. LO: 11.9

To be effective, nurse managers should focus on which area? Cost-effective operation of the unit Motivation of staff Accomplishing organizational goals The patients and their needs

Answer: dAll of the options are considerations of an effective manager. However, at the core of all nursing management actions must be the benefit of the patient or patient population that is served. LO: 12.3

The nurse is caring for a 16-year-old boy receiving chemotherapy for testicular cancer. He says that his parents are religious and left a cross next to his bed for "good luck." What is the most appropriate response by the nurse? a. "Would you like to talk with a chaplain?" b. "Sounds like you are not very religious." c. "How well do you get along with your parents?" d. "What helps you get through tough times?"

Answer: dAsking an open-ended question is the best response that the nurse can make to this patient's comment. It will encourage the patient to share what he thinks would be most supportive at this time. Asking the patient if he would like to talk with a chaplain is incongruent with his comment about having a cross for good luck. Sharing that he does not sound religious is judgmental and asking about his relationship with his parents is changing the subject. LO: 22.7

Which direct-care intervention would be most effective in helping a patient cope emotionally with a new diagnosis of cancer? a. Reassessing for changes in the patient's physical condition b. Teaching the patient various methods of stress reduction c. Referring the patient for music and massage therapy d. Encouraging the patient to explore options for care

Answer: dEncouraging the patient to explore options for care empowers the patient to have some control over the situation and to be actively involved in care planning. It is a form of informal counseling. Reassessment and teaching are not immediately indicated at this time. Although referring a patient with a new cancer diagnosis may be helpful, it is an indirect care intervention. LO: 9.2

. Which statement illustrates a characteristic of goals within the care planning process? a. Goals are vague objectives communicating expectations for improvement. b. Short-term goals need not be measurable, unlike long-term goals. c. Goal attainment can be measured by identifying nursing interventions. d. Long-term goals are helpful in judging a patient's progress.

Answer: dLong-term goals are very useful in determining patient progress. Both short-term and long-term goals need to be measurable. Goal attainment is based on patient actions, not nursing actions.

Which statement best describes the relationship of medical diagnoses and nursing diagnoses? a. Medical diagnoses are imbedded in nursing diagnoses. b. Nursing diagnoses are derived from medical diagnoses. c. Medical diagnoses are not relevant to nursing diagnoses. d. Medical diagnoses may be interrelated to nursing diagnoses.

Answer: dNursing diagnoses consider the underlying etiology, needs, potential concerns, and patient response to a patient's medical diagnosis; thus, the two types of diagnoses are interrelated. Medical diagnoses are not imbedded in nursing diagnoses and nursing diagnoses are not derived from medical diagnoses, because that would limit the scope of assessment and care that is provided for patients. Nurses consider the medical diagnosis as one aspect of concern when identifying an existing or potential health problem and the patient's response; thus, medical diagnoses are relevant but not the focus of nursing diagnoses. LO: 7.6

What is the most important reason for nurses to use a standardized taxonomy, such as the ICNP, CCC, or NANDA-I? a. Insurance documentation b. Professional autonomy c. EMR data analysis d. Patient safety

Answer: dSafety is the most important reason for using standardized language to communicate a patient's needs and information. Using the same definitions of terms helps nurses and other health care professionals interpret the information. Helping with insurance documentation, supporting professional autonomy, and EMR data analysis are uses for ICNP, CCC, or NANDA-I taxonomy, but they are not the most important. LO: 7.1

What is the most important action for a nurse to take to have a new nursing diagnosis considered for inclusion in the ICNP or NANDA-I taxonomies? a. Share concerns with the nurse manager on the nursing unit. b. Offer alternative care for a patient and family members. c. Discuss how to address patient needs with physicians. d. Provide evidence-based research to support nursing care.

Answer: dSupporting a suggestion for a new nursing diagnostic label with research is required for consideration by both ICNP and NANDA-I. Sharing concerns, providing alternative care, and advocating for patients are all a part of the nursing role but are not the most important part of having a diagnosis considered for inclusion in the ICNP or NANDA-I taxonomies. LO: 7.2

. In Swanson's Caring Theory, the nurse demonstrates caring using several techniques. Which action is appropriate and most important for the nurse to include in a patient's plan of care? a. Call patients by their first name to demonstrate a caring attitude. b. Sit at the bedside for at least 5 minutes each hour. c. Use touch based on the nurse's judgment of what is appropriate. d. Ask the patient to identify the most important thing to accomplish during the nurse's shift.

Answer: dThe answer is based on the idea that the patient should always, whenever possible, be included in developing the plan of care and especially in setting the patient's own goals. The other three answers are close, but something is wrong with each one. Calling patients by their first name to demonstrate a caring attitude is incorrect, because the nurse should call each patient by the individual's preferred name. Sitting at the bedside for at least 5 minutes each hour is incorrect, because the nurse should try to sit at the bedside for 5 minutes each shift rather than each hour. Using touch based on the nurse's judgment is wrong, because the nurse should allow the patient to decide how much touch is appropriate. LO: 2.6

The nurse manager is monitoring overtime for the unit. She closely monitors staff hours and does not allow staff to come in for extra hours if they are over their allotted time per week. This is an example of which of Mintzberg's decisional roles of the manager? Entrepreneur Disturbance handler Negotiator Resource handler

Answer: dThe resource handler function of the decisional role of the manager includes monitoring the budget and regulating the use of personnel time. In the entrepreneur role, the manager is the problem solver. The disturbance handler responds to unplanned change. The negotiator works within and outside of the organization to intercede for resources and help. LO: 12.3

What should the nurse consider before implementation of all nursing interventions? (Select all that apply.) a. Potential communication barriers b. Diverse cultural practices c. Scope of nursing practice d. Functional status of the patient e. Time of most recent shift change

Answers: a, b, c, d Cultural practices, functional status, communication barriers, and scope of practice influence whether an intervention should or may be implemented. Shift change time is not necessary to consider before implementation of most interventions. LO: 9.1

What should be taken into consideration by the nurse when deciding on interventions to include in a patient's plan of care? (Select all that apply.) a. Patient's treatment preferences b. Cultural and ethnic influences c. Nurse's professional expertise d. Current evidence-based research e. Convenience to the nursing staff

Answers: a, b, c, d Patient treatment preferences, cultural and ethnic influences, the level of a nurse's professional expertise, and current evidence-based research should all be taken into consideration when planning care. The convenience to nursing staff should not be of concern. LO: 9.2

Which factors influence whether a message is effectively communicated? (Select all that apply.) a. Timing of the conversation b. Educational level of participants c. Mode of communication used d. Physical environment of discussion

Answers: a, b, c, d Timing of a conversation dramatically influences the receptivity of the receiver. The educational level of those seeking to communicate has an impact on the type of language and technical terminology that can be used in conversation. Using more than one mode of communication can enhance the effectiveness of a message. Making sure the environment is devoid of excess noise and distraction can facilitate a greater understanding of shared information. LO: 3.1

Which behaviors are expected of the nurse at the leadership informatics competency level? (Select all that apply.) Collect accurate assessment data. Conduct informatics research. Group assessment data. Document data appropriately on the electronic health record (EHR). Integrate information science into nursing practice.

Answers: a, b, c, d, e The nurse at the leadership level of informatics competency addresses ethical and management issues related to using IT in nursing practice, education, research, and administration. Informatics research and integration of the sciences would be within the leadership competency. The nurse at this level is also proficient in all lower levels of informatics competency; thus, the nurse can see data relationships and is able to collect and group data. The nurse is skilled in the use of computer technology and can document in the EHR.

Which of the following would be examples of positive reinforcers? (Select all that apply.) a. Clapping for a child who successfully used the toilet b. Offering praise for a child who behaved in a store c. Taking a child to the playground after a positive teacher report d. Placing a child in time-out for misbehavior. e. Stickers for positive play with peers.

Answers: a, b, c, e Clapping, offering praise, and giving rewards are all positive reinforcements that are likely to increase desired behavior(s). Time-out is a type of negative punishment used to discourage unwanted behavior. LO: 17.1

Which factor(s) is/are likely to influence the transition from adolescence to adulthood? (Select all that apply.) a. Cultural beliefs b. Societal values c. Personal beliefs and expectations d. Governmental rules e. Peer influences

Answers: a, b, c, e Cultural beliefs, societal values, personal beliefs and expectations, and peer influences are all factors that influence an individual's transition into adulthood. Although governmental rules may influence certain societal values and beliefs, they are not directly responsible for the transition through adulthood. 18.3

Which are reasons that accurate documentation in the medical record is important? (Select all that apply.) Reimbursement for care Evidence of care provided Communication between health care providers Non-legal documentation of a nurse's actions Promotion of continuity of care

Answers: a, b, c, e Documentation in the medical record is important for reimbursement for care, for providing a record of services, for communication between providers, and for promoting continuity of care. The record is a legal document, not a non-legal document. LO: 10.2

A profession has specific characteristics. In regard to how nursing meets these characteristics, which criteria are consistent and standardized processes? (Select all that apply.) a. Code of ethics b. Licensing c. Body of knowledge d. Educational preparation e. Altruism

Answers: a, b, c, e Nursing as a profession has a code of ethics, licensing, a body of knowledge, and altruism. Because there are multiple paths of education for nursing and not a standard entry into practice, this is one criterion of a profession that varies in nursing.

Which items are supported by point-of-care use of information technology? (Select all that apply.) More accurate documentation Direct access to diagnostic results Confidentiality Direct access to records by patients Access to medication profiles

Answers: a, b, c, e Point-of-care use of information technology provides more accurate documentation because the nurse documents patient information in real time. Diagnostic results are immediately available to the nurse. Confidentiality is protected when the nurse documents at the bedside using a secure log-in and password. Medication profiles are available to the nurse at bedside. Patients can request medical records through a process but do not have direct, immediate access to their medical records because these records are secured by log-ins and passwords accessible only to health care providers caring for each patient. LO: 15.1

. When an injury to a child is suspicious for abuse, which is/are important to document? (Select all that apply.) a. Size and location of bruising b. Distinguishing characteristics of injuries c. Height and weight of the child d. Time of last meal e. General state of health of the child

Answers: a, b, c, e The size and location of the bruising, distinguishing characteristics of injuries, height and weight of the child, and the general state of health of the child all are considerations in determining possible child abuse or neglect. The exact location and the nature of the injury are important to document. A child who has been abused over time may be small and have other health problems stemming from the abuse. The time of the last meal may be important information but usually is not related to suspected abuse. LO: 17.6

What situations would necessitate modification of a patient's plan of care? (Select all that apply.) a. Decrease in patient's level of orientation b. Discharge of patient to rehabilitation facility c. Patient adherence to established plan of care d. Sudden onset of shortness of breath in patient receiving oxygen

Answers: a, b, d A decrease in level of orientation, discharge to a rehabilitation facility, and sudden onset of shortness of breath, even if a patient is already receiving oxygen, are situations in which the plan of care needs to be modified to address the patient's changing needs. Patient adherence to the plan of care implies that the plan is working; therefore, it does not need to be revised at this time. LO: 9.9

Which of the following questions are appropriate to ask during a transcultural assessment? (Select all that apply.) a. How do you act when you are angry? b. What is your role in your extended family? c. Why do you continue to speak German at home? d. When communicating with friends, how close do you stand? e. What is the purpose of not preparing beef with milk products?

Answers: a, b, d How a person acts when angry, the person's role in the family, and comfort with proximity all are relevant aspects of the patient's cultural norms according to Giger and should be assessed to raise the nurse's awareness of patient needs. Asking patients why they use their native language in the home is unnecessary. If primary language information is needed, the nurse should simply ask what language is spoken in the home. The nurse should not try to seek information about the reason a person maintains dietary traditions during the assessment process.

The nurse is assessing data for a group of older adults. Which patient(s) in this group is/are exhibiting normal signs of aging? (Select all that apply.) a. The patient with knee pain and wrinkles around the eyes b. The patient who needs reading glasses and states that the food tastes bland c. The patient who is confused and does not know the current year d. The patient who states that constipation is an increasing problem e. The patient who is showing signs of depression and hopelessness

Answers: a, b, d Normal aging includes signs of decline in many organ systems. Knee pain, skin wrinkles, need for reading glasses, decline in taste buds, and constipation all can occur with aging. Normal aging does not include dementia or depression, even though these can both be diagnosed in the older adult population. LO: 18.7

Which factors affect the nursing shortage? (Select all that apply.) a. Aging faculty b. Increasing elderly population c. Job satisfaction due to adequate number of nurses d. Aging nursing workforce e. Greater autonomy for nurses

Answers: a, b, d The nursing workforce and nursing faculty are aging. The entire population is aging, which increases the need for more nurses. The insufficient number of nurses leads to job dissatisfaction and burnout. Greater autonomy for nurses does not affect the nursing shortage.

Which actions describe the primary scope of practice of a public health nurse? (Select all that apply.) a. Assesses populations within the community. b. Emphasizes health promotion and disease prevention using evidence-based strategies. c. Focuses on individual, setting-specific care. d. Collaborates with community stakeholders to ensure that essential health services are available to the community. e. Promotes health policies to protect health.

Answers: a, b, d, e Public health nurses focus primarily on population-focused practice as opposed to individually focused care. Population-focused care is aimed at health promotion and disease prevention. Assessment, policy development, and ensuring that essential services are provided are the three core functions of public health.

If the nurse chooses the Nursing Outcome Classification (NOC) Appetite for a chemotherapy patient, which outcome indicators would be acceptable for evaluation of goal attainment? (Select all that apply.) a. Expressed desire to eat b. Report that food smells good c. Use of relaxation techniques before meals d. Preparation of home-cooked meals for self and family e. Use of nutritional information on labels to guide selections

Answers: a, b, dSharing a desire to eat, reporting that food smells good, and preparing meals are indications of an increased appetite. Although relaxation techniques may decrease anxiety associated with eating, they do not indicate an increase in appetite. Reading nutrition labels is unlikely to increase a person's appetite. LO: 8.4

Which statements describe a component discussed in nursing theories? (Select all that apply.) a. Optimal functioning of the patient b. Interaction with components of the environment c. The conceptual makeup of the administration of the hospital d. The illness and health concept e. Safety aspect of medication administration

Answers: a, b, dThere are four components that a nursing theory discusses: (1) the patient, (2) health, (3) environment, and (4) nursing. Hospital administration and safe medication administration, while important to nursing, are not part of nursing theories.

Which are included in the ANA Standards of Professional Nursing Practice? (Select all that apply.) a. Standards of professional performance b. Code of ethics c. Standards of practice d. Legal scope of practice e. Licensure requirements

Answers: a, c ANA standards have two parts: One is standards of professional performance and the other is standards of practice. ANA has a separate document that is a code of ethics. Nurse practice acts are a legal scope of practice. Licensure requirements include graduation from an accredited school of nursing and passing a licensure exam.

Which entities or documents specifically address the role of the nurse in research? (Select all that apply.) a. American Nurses Association (ANA) standards of practice b. Institutional review board (IRB) c. Hospital Magnet status d. The Joint Commission (TJC) e. MD Consult

Answers: a, c The American Nurses Association (ANA) standards of professional performance support the nurse's role in research, and hospital Magnet status includes the nurse using evidence to improve quality of care. The Joint Commission (TJC) does not address nursing research, institutional review boards approve all research involving human subjects if it is funded by the federal government, and MD Consult is a background resource for research. None of these three entities specifically address the role of the nurse in research. LO: 13.1

Which statements reflect the practice of transcultural nursing? (Select all that apply.) a. May be considered a general and specialty practice area b. Focuses on the worldview rather than patient needs c. Challenges traditional ethnocentric nursing practice d. Aims to identify individual patient care preferences e. Focuses patient care on the nurse's cultural norms

Answers: a, c, d Transcultural nursing is a general and specialty practice that focuses on both worldview and individual patient and family needs for planning and providing care. It challenges nurses to investigate other cultures in order to reject ethnocentric care and respond to individual needs. LO: 21.4

What legal consequences may a nurse experience if the nurse is convicted of a crime? (Select all that apply.) a. Loss of nursing licensure b. Employment affirmation c. Monetary penalty d. Unit transfer e. Imprisonment

Answers: a, c, e Nurses who are convicted of a crime may have their nursing licenses revoked, be required to pay fines or pay restitution to patients or their families for damages, and be incarcerated for various periods, depending on the severity of the crime. Nurses who commit crimes are most often placed on probation pending the outcome of court proceedings or dismissed from their employment, not affirmed or transferred to work on another unit. LO: 11.7

The nurse cares for dying patients and understands that "nearing death awareness" is a phenomenon evident by which patient statement(s)? (Select all that apply.) a. "Where are my shoes? I need to get ready for the trip." b. "Is my daughter from California going to come and visit before I die?" c. "When do you think that I am going to die?" d. "I was just talking to my daughter (deceased)." e. "How much longer can I live without food or water?"

Answers: a, d Nearing death awareness has been described as a state manifested by a special communication of the dying that may occur in patients who are approaching death or are imminently dying. People experiencing this "nearing death awareness" may appear confused, but they may actually be making the transition from life to death. All of the other options are questions that dying people may ask, but they do not represent nearing death awareness. LO: 42.7

The nurse assigned to the neonatal intensive care unit (NICU) has spent most of the day working with a critically ill infant, with the mother standing by. The infant experiences a cardiac arrest and does not survive. The mother spends an hour crying and holding the baby, saying good-bye. Which spiritual care interventions are most appropriate for the nurse to implement? (Select all that apply.) a. If desired, briefly hold the baby to say good-bye after the mother leaves. b. Follow procedures to prepare the body for transport to the morgue. c. Visit the mother the next day to see how she is doing. d. Call the family's spiritual adviser or the chaplain. e. Ask the mother if you could call a family member or friend to be with her.

Answers: a, d, e It is important for nurses to take time to say "good-bye" to patients with whom they have developed a relationship. In this case, it would be appropriate for the nurse to hold the infant briefly, if desired, after the family has left before preparing the body for the morgue. With consent, the mother needs to be surrounded by appropriate persons to provide spiritual support, including a chaplain, family members, and friends. Although it is critical that the nurse follow procedures in preparing bodies for the morgue, it is not considered spiritual care. Visiting the mother after her loss could be viewed as a violation of professional boundaries, especially for a nurse who worked with the family for only a day. LO: 22.7

Which actions are part of the evaluation step in the nursing process? (Select all that apply.) a. Recognizing the need for modifications to the care plan b. Documenting performed nursing interventions c. Determining whether nursing interventions were completed d. Reviewing whether a patient met the short-term goal e. Identifying realistic outcomes with patient input

Answers: a, dRecognizing the need for care plan modifications and determining whether a goal or outcome is met are actions that take place during evaluation. Making sure interventions are completed and documenting them are part of implementation. Identifying outcome criteria is done during the planning stage of the nursing process. LO: 9.8

Which intrinsic factors would be of major concern to the nurse when the community has an outbreak of pertussis? (Select all that apply.) a. Age b. Gender c. Ethnic group d. Cultural background e. Immunization status

Answers: a, e Age and immunization status would be of most concern because people who have not had a pertussis vaccine could be a source of potentially fatal infection in infants. In the case of pertussis vaccination, gender, ethnicity, and cultural background are not of particular concern. LO: 23.3

Patients are participating in a study to identify genetic disorders. About which potential concerns should the nurse researcher be aware? (Select all that apply.) a. Violation of confidentiality if a disorder is revealed b. Possible adverse consequences related to employment c. Possible adverse consequences related to reputation d. Possible adverse consequences to insurability e. Inability to prevent the progression of genetic disorders

Answers: a, e Potential concerns include violation of confidentiality and the inability to prevent progression of genetic disorders. Employment, reputation, and insurability would not be of concern as a result of participating in the study. LO: 13.3

Which descriptions are advantages of health care information technology (IT)? (Select all that apply.) Increases health care delivery costs Improves communication among providers Improves administration functions Increases the potential for errors Decreases the safety of providing care

Answers: b, c Health care IT improves communication among providers by providing immediate access that is legible and standardized to patient data by all providers. IT improves administrative functions by addressing the issues of quality, cost-effectiveness, and outcomes of care. Although there are initial costs to purchase hardware and software, these systems are cost-effective in the long run. Systems that support data collection at the point of care can directly enhance patient care by decreasing the potential for errors, especially in interpreting handwriting, and supporting improved assessment and data communication. LO: 15.1

NCLEX Client Needs: Physiological Integrity: Pharmacological and Parenteral Therapies 4. Nursing students are held to which standards by the Code of Ethics for Nurses? (Select all that apply.) a. Clinical skills performance equal to that of an experienced nurse b. Demonstration of respect for all individuals with whom the student interacts c. Avoidance of behavior that shows disregard for the effect of those actions on others d. Accepting responsibility for resolving conflicts in a professional manner e. Incorporating families in patient care regardless of patient preference

Answers: b, c, d Nursing students are expected to demonstrate respect, avoid hurting others by their actions, and take responsibility for resolving conflicts in a professional manner, much the same as professional nurses. Student nurses are not required to perform clinical skills at the level of expertise exhibited by an experienced nurse. Involving a patient's family in care without the patient's approval indicates a lack of respect for patient autonomy. LO: 11.3

. Which nursing interventions would be necessary in caring for a patient with cognitive alterations who is hospitalized? (Select all that apply.) a. Apply wrist restraints for combativeness. b. Place a clock in the room for orientation. c. Keep floor free of clutter for safety. d. Identify staff with each interaction. e. Play loud music for distraction.

Answers: b, c, d Reality orientation is important for patients with cognitive alterations. Keeping the floor free of clutter prevents falls. All staff members should wear a readily visible name tag and state their name and what they are going to do. Soft music and dim lights will create a less distracting environment for the patient. Restraints may cause increased confusion and agitation and are used in special circumstances only. LO: 31.6

. Which factors contribute to the nurse having difficulty keeping up with the latest research findings? (Select all that apply.) a. Implementation delays b. Proliferation of research c. Volume of health care literature d. Hours spent in direct patient care e. The need to read three articles every day of the week

Answers: b, c, d The proliferation of research has led to huge volumes of literature, which would require the nurse to read an unrealistic number of articles daily to remain current. Hours spent in direct patient care make it nearly impossible to keep up with current practices. Implementation delays affect the nurse's ability to put alternative interventions into practice rather than keep up with research. LO: 13.4

Which attributes are important in nursing documentation? (Select all that apply.) Inconsequentiality Timeliness Relevancy Accuracy Factual basis

Answers: b, c, d, e Documentation should be completed in a timely manner, be relevant and concise, and be accurate and factual. Inconsequentiality suggests a lack of importance; documentation is an important part of patient care and nursing responsibility. LO: 10.3

Which interventions can the nurse initiate independently while providing patient care? (Select all that apply.) a. Ordering a blood transfusion b. Auscultating lung sounds c. Monitoring skin integrity d. Applying heel protectors e. Adjusting antibiotic dosages

Answers: b, c, dAuscultating lung sounds and monitoring skin integrity are both important aspects of basic patient assessment that are required independent nursing actions. Ordering and applying heel protectors is done independently by nurses to prevent skin breakdown on patients confined to the bed. Ordering blood transfusions and adjusting antibiotic dosages are the responsibility of the patient's primary health care provider. LO: 8.6

A nurse is working with a 35-year-old patient who needs to decide whether to donate a kidney to his brother, who has been in renal failure for 5 years. The patient shares with the nurse that the decision is especially difficult because he would not be able to continue to work in his current profession and would be unable to support his three small children if he ever needed dialysis. Which interventions would be most appropriate for the nurse to implement in this situation? (Select all that apply.) a. Explain that it is unlikely that he will ever need dialysis even if he has only one kidney. b. Guide the patient through a values clarification process to help him make a decision based on his values. c. Provide information the patient needs to help him make an informed decision. d. Ask for his permission to contact the kidney donation team to answer any questions he may have. e. Assure him that everything will be alrig

Answers: b, c, dEncouraging the patient to make a decision based on his personal values, providing necessary information, and offering consultation with individuals most familiar with the kidney donation process are all excellent interventions. It would be impossible to predict whether a patient will need dialysis in the future, making this type of statement misleading. Assuring a patient that everything will be alright is false reassurance that is nontherapeutic and should be avoided in all situations. LO: 2.3

Which nursing diagnosis statements are appropriately written according to the 2021-2023 NANDA-I format? (Select all that apply.) a. Risk for Infection related to elevated temperature and white blood count b. Readiness for Effective Family Process as evidenced by an expressed desire for improved communication and mutual respect verbalized by family members c. Impaired health maintenance related to inability to access care as evidenced by failure to keep appointments, homebound status d. Risk for Hemorrhaging as evidenced by prolonged clotting time e. Chronic Pain related to osteoarthritis as manifested by verbalized postoperative discomfort

Answers: b, c, dReadiness for Effective Family Process is a health promotion nursing diagnosis and is written with two sections: the label and the defining characteristics. Impaired health maintenance is a problem-focused nursing diagnosis that requires a related factor and defining characteristics. Risk for Hemorrhaging requires at least one risk factor, which it has as it is written. Use of related factors in a risk nursing diagnosis is not the accepted NANDA-I format. The nursing diagnosis of Chronic Pain is incorrectly written because it includes a medical diagnosis and a related factor that is supportive of acute rather than chronic pain. LO: 7.4

A 61-year-old man is undergoing an emergency cardiac catheterization. The nurse gives his wife the registration paperwork to complete. Which observed actions may indicate a health literacy issue? (Select all that apply.) Putting on glasses before beginning the paperwork. Asking someone in the waiting area to read the forms to her. Waiting until her daughter arrives to begin the paperwork so that her daughter can complete the forms. Setting the clipboard aside and staring tearfully out the window. Returning the forms only partially filled out, with missing or inaccurate information.

Answers: b, c, e Asking someone else to read the form, waiting for help with the forms, and partially or inaccurately filling out forms are behaviors indicative of potential health literacy issues. Needing glasses does not correlate directly with health literacy. A tearful spouse requires additional assessment to see whether health literacy is a problem. The wife may be overwhelmed and feel unable to complete the forms, or she may need to collect her thoughts in the midst of a stressful time. LO: 14.7

What signs and symptoms would the nurse appropriately cluster as supporting data for a patient with extreme anxiety? (Select all that apply.) a. Denies any difficulty falling asleep b. Elevated pulse rate, auscultated at 140 bpm c. Continuous foot tapping throughout intake interview d. Demonstrates how to give insulin self-injection without hesitation e. Patient states, "I feel nervous all the time, especially when I am alone."

Answers: b, c, eAn elevated pulse rate, continuous toe tapping, and verbalizing nervousness are consistent with extreme anxiety and should be clustered together. Ease of falling asleep and being able to focus on a challenging task, such as giving an injection, are not indicative of a patient experiencing a high level of anxiety.

When caring for a postoperative patient on a surgical unit, which nursing intervention(s) are included in the patient's plan of care to help reduce the risk of both pulmonary embolism (PE) and deep vein thrombosis? (Select all that apply.) a. Encourage oral hydration. b. Apply antiembolism stockings. c. Monitor laboratory values for clotting times. d. Encourage frequent ambulation. e. Instruct the patient to perform the Valsalva maneuver frequently.

Answers: b, d Nursing care to prevent clots from developing and preventing thrombus from becoming detached include use of antiembolism stockings, sequential compression devices, leg exercises, and ambulation to promote circulation. Encouraging oral hydration helps liquefy secretions and prevent pneumonia. Monitoring laboratory values should be done if a patient is receiving anticoagulants, but it does not help prevent clots. The patient should be instructed not to perform the Valsalva maneuver, because any activity that increases intrathoracic pressure increases the risk of an emboli traveling to the pulmonary artery. LO: 37.8

Which step(s) can nurses and health care providers take to remove barriers to identifying and treating victims of domestic violence? (Select all that apply.) a. Call the police. b. Ask about abuse. c. Ask for proof of domestic violence. d. Screen for domestic violence with all patients. e. Disregard reported abuse in spouses.

Answers: b, d Screening for domestic violence with all patients and asking about abuse will help identify patients who may be victims. Calling the police is a step that may be taken when abuse is confirmed but is not a step in identifying victims. Taking this step will be decided on with the involvement of both health care providers and the patient. Asking for proof may cause the victim to mistrust the health care provider. Never disregard a report of abuse. Abusers often are related to or married to the victim. LO: 18.4

When teaching a patient about surgical procedures for birth control, the nurse would include which methods? (Select all that apply.) a. Diaphragm b. Vasectomy c. Hormone injection d. Tubal ligation e. Oral contraceptives

Answers: b, d Vasectomy and tubal ligation are surgical procedures for birth control. The diaphragm is a mechanical barrier, oral contraceptives are absorbed through the gastrointestinal tract, and hormones in injections are absorbed systemically. LO: 24.5

For which reasons are patients unlikely to introduce the topic of sex with health care providers? (Select all that apply.) a. Most patients have few questions or problems relating to this topic. b. They are too embarrassed to discuss the topic of sex with a health care provider. c. Female patients prefer to discuss problems with female health care providers. d. They assume that health care professionals know little about sexual functioning. e. They are concerned that the health care professionals will be judgmental.

Answers: b, eEmbarrassment to discuss a personal subject and fear of being judged can cause the patient to avoid introduction of the topic. Gender is not a consideration in this circumstance, and it cannot be generalized that patients have few questions or problems on any topics. Health care professionals are generally considered to be knowledgeable in subject matters associated with health and illness. LO: 24.9

Which nursing diagnoses are used in developing a patient teaching plan? (Select all that apply.) Moral Distress Lack of Knowledge Difficulty Coping Teaching about Disease Anxiety

Answers: bLack of Knowledge and Literacy Problem are appropriate nursing diagnoses for use in developing a patient teaching plan. Moral Distress is a nursing diagnosis for those facing ethical decisions. Difficulty Coping is not a nursing diagnosis used in developing a teaching plan, but if a patient is not coping effectively, it may affect the ability to learn. A nursing diagnosis of Anxiety may affect the patient's ability to learn but is not directly related to developing a teaching plan. Teaching about Disease is not a nursing diagnosis. It is an intervention perform

A patient is on the way to the hospital in an ambulance and is asked to participate in a research protocol for a new treatment for myocardial infarction. What is this an example of? (Select all that apply.) a. Standard practice b. An ethical dilemma c. A violation of informed consent d. A patient who is in a vulnerable population category e. Compliance with important ethical issues of justice and autonomy

Answers: c, d Because the patient does not have time to consider the option, this is a violation of informed consent. Because the patient is in an emergency situation, the patient is in a vulnerable population category. It is neither standard practice nor an ethical dilemma because research protocols clearly indicate when a person is to be approached for participation in research studies. Asking a patient to participate in research in an emergency situation is a violation of the ethical issues of justice and autonomy. LO: 13.3

A new nurse is about to insert a nasogastric (NG) tube for the first time but is not sure what equipment to gather or how to begin the procedure. The patient is an 80-year-old woman who is frightened and slightly confused. Which actions by the nurse would best demonstrate caring? (Select all that apply.) a. Offer the patient pain medication to help her calm down. b. Hold the patient's hand while inserting the NG tube. c. Speak calmly while explaining the procedure to the patient beforehand. d. Ask another, more experienced nurse for assistance before initiating care. e. Delay inserting the NG tube until the patient's husband comes to visit.

Answers: c, dUsing a calm voice and seeking help from an experienced nurse exhibit caring for the patient and will help allay patient anxiety. Medicating a patient for pain before the experience will not automatically alleviate patient anxiety and may cause the patient to experience greater confusion. The nurse will need to use both hands to safely insert the NG tube and promote a positive patient outcome; thus, the nurse is unable to hold the patient's hand during the procedure. Delaying insertion of the NG tube may make the patient's symptoms worse, which does not demonstrate caring.

Several theorists have identified stages of the grieving process. The nurse understands these stages and knows that people progress through them in an individualized manner. Which statements are true regarding the steps of the grieving process? (Select all that apply.) a. There is a definite "timetable" or period of time specific to each stage of the grieving process. b. Nursing interventions are generalized across all stages of the grieving process. c. Tasks to be achieved at each stage have been identified by each theorist. d. There is a common stepwise progression through each stage of the grieving process. e. Not all individuals will experience all stages of grief.

Answers: c, e Each stage of the grieving process has associated tasks that allow successful grieving to occur on an individualized basis. Theories that describe the grieving process are simply guides to understanding the process of grief, and there is no specific timeline regarding when people "should be" in a certain stage, "should" move from one stage to the next, or follow a stepwise progression. Not all people will experience all stages of grief. Essentially, there is no timetable for the process of grief and bereavement. Nurses need to understand these stages and the feelings and emotions common to each stage. This facilitates nursing interventions that can be focused on the stage the patient is experiencing or the task the person is attempting to complete related to the process of grief. LO: 42.1

b. Moro reflex

In response to a loud sound, babies will extend their arms, throw back their head, and often cry.

d. Tonic neck reflex

When an infant is lying in a supine position and the head is turned to the side, the infant will extend the arm on the side the head is facing. The opposite arm will be bent.

c. Rooting reflex

When the corner of a baby's mouth is touched, the baby will turn the head toward the touch and open the mouth.

a. Sucking reflex

When the roof of a baby's mouth is touched, the baby will begin to suck

Babinski reflex

When the sole of the infant's foot is stroked, the infant's big toe moves upward, while the remaining toes fan outward.

After analyzing patient data, the registered nurse (RN) on an inpatient medical unit delegates vital signs and morning care to the unlicensed assistive personnel (UAP) for five stable patients. The nurse asks the UAP to document the vital signs and report any abnormal results immediately. Which rights of delegation is the nurse demonstrating? (Select all that apply.) Right person Right circumstance Right time Right supervision Right patient

a, b, d The nurse is demonstrating all of the rights of delegation. Right time and right patient are not part of the Five Rights of Delegation. They are a part of the Rights of Medication Administration. The Five Rights of Delegation are: 1. Right task 2. Right person 3. Right circumstance 4. Right communication 5. Right supervision

Which actions does the nurse need to take before determining the types of nursing diagnoses that are applicable to a patient? (Select all that apply.) a. Review the patient's past and present medical history. b. Analyze the nursing assessment data to determine whether information is complete. c. Outline an individualized plan of care to address each concern. d. Consider potential complications to which the patient is susceptible. e. Evaluate how the patient has responded to treatment.

e. Evaluate how the patient has responded to treatment. Answers: a, b, dBefore determining the types of nursing diagnoses that are appropriate for a patient, the nurse must review and analyze all of the patient's data, including the medical history, for completeness and accuracy. Considering the vulnerability of a patient to potential complications permits the nurse to identify the need for risk nursing diagnoses. Outlining an individualized plan of care takes place during the planning stage of the nursing process after the nursing diagnoses have been identified. Evaluation of a patient's response to treatment is part of the evaluation stage of the nursing process. LO: 7.5

Which activity is important to include in the plan of care for a patient with a peripherally inserted central catheter (PICC)? (Select all that apply.) Change the PICC dressing only when it becomes soiled or loose. Change the IV tubing every 5 to 7 days. c. Take blood pressure in the arm without the PICC line. d. Use only macrodrip tubing with IV infusions through the PICC line. e. Use alcohol-impregnated disinfection caps on needleless ports when not in use.

e. Use alcohol-impregnated disinfection caps on needleless ports when not in use. Answer: c, e PICC dressings should be changed every 5 to 7 days regularly using sterile technique for transparent dressings and gauze dressings must be changed every 48 hours. This is to keep the site sterile and prevent central line associated blood stream infections (CLABSIs). Dressings should also be changed whenever wet, soiled, or loose. PICC IV tubing is usually changed every 24 hours depending on facility policy. Never take blood pressure in an arm with a PICC. Macrodrip or microdrip tubing can be used for infusions through a PICC. Green or orange disinfection caps are alcohol impregnated and should be used to keep ports clean when not in use. LO: 39.5


Conjuntos de estudio relacionados

Texas Government (2306)- The Executive Branch-Ch 4

View Set

Pharmacology Chapter 36- Antihistamines, Decongestants, Antitussives, and Expectorants

View Set

Chapter 9 Rehabilitation and Restorative Care

View Set

Chapter 10: Investment Company Products

View Set

Conduction psych research in the real world

View Set